You are on page 1of 201

Q1 p ©

You review a 42-year -old woman six weeks following a renal transplant
for focal segmental glomerulosclerosis. Following the procedure she
was discharged on a combination of tacrolimus, mycophenolate, and
prednisolone. She has now presented with a five day history of feeling
generally unwell with anorexia, fatigue and arthralgia. On examination
her sclera are jaundiced and she has widespread lymphadenopathy with
hepatomegaly. What is the most likely diagnosis?

Hepatitis C

Epstein-Barr virus

HIV

Hepatitis B

Cytomegalovirus

Submit answer

Reference ranges v

Score: 0%

CamScanner
fid) '?' ® £ JI I C> IAJ ISLa
^
You review a 42-year-old woman six weeks following a renal transplant
for focal segmental glomerulosclerosis. Following the procedure she
was discharged on a combination of tacrolimus, mycophenolate, and
prednisolone. She has now presented with a five day history of feeling
generally unwell with anorexia, fatigue and arthralgia. On examination
her sclera are jaundiced and she has widespread lymphadenopathy with
hepatomegaly. What is the most likely diagnosis?

Hepatitis C

Epstein-Barr virus

HIV

Hepatitis B

Cytomegalovirus

Cytomegalovirus is the most common and important viral


infection in solid organ transplant recipients
Important for me Less important

Ganciclovir is the treatment of choice in such patients.

A f p Discuss (3) Improve

Next question >


Renal transplant: HLA typing and graft failure

The human leucocyte antigen (HLA) system is the name given to the
major histocompatibility complex (MHC) in humans. It is coded for on
chromosome 6.

CamScanner
major histocompatibility complex (MHC) in humans. It is coded for on
chromosome 6.

Some basic points on the HLA system


• class 1 antigens include A, B and C. Class 2 antigens include
DP,DQ and DR
• when HLA matching for a renal transplant the relative importance
of the HLA antigens are as follows DR > B > A

Graft survival
• 1 year = 90%, 10 years = 60% for cadaveric transplants
• 1 year = 95%, 10 years = 70% for living-donor transplants

Post-op problems
• ATN of graft
• vascular thrombosis
• urine leakage
• UTI

Hyperacute acute rejection (minutes to hours)


• due to pre-existent antibodies against donor HLA type 1 antigens
(a type II hypersensitivity reaction)
• rarely seen due to HLA matching

Acute graft failure (< 6 months)


• usually due to mismatched HLA. Cell-mediated (cytotoxic T cells)
• other causes include cytomegalovirus infection
• may be reversible with steroids and immunosuppressants

Causes of chronic graft failure (> 6 months)


• both antibody and cell mediated mechanisms cause fibrosis to
the transplanted kidney (chronic allograft nephropathy)
• recurrence of original renal disease (MCGN > IgA > FSGS)

CamScanner
Q Q4 P

A 38-year-old woman presents for review of her hypertension. She was


diagnosed 6 months ago, her blood pressure has been refractory to a
combination of amlodipine and ramipril. On examination, heart rate is
80bpm and blood pressure is 170 / 100mmHg.

Investigations:

Serum potassium 2.9 mmol/ L (3.5- 5.0)

Plasma aldosterone (after 30 minutes supine) 600 pmol/ L (135-400)

Plasma renin activity (after 30 minutes supine) 6.8 pmol/ mL/ hr (1.1-2.7)

What is the most likely cause of her hypertension?

Bartter syndrome

Bilateral renal artery stenosis

Phaeochromocytoma

Pregnancy

Primary hyperaldosteronism

Submit answer

Reference ranges

CamScanner
Bilateral renal artery stenosis

Phaeochromocytoma

Pregnancy

Primary hyperaldosteronism

Primary hyperaldosteronism, bilateral renal artery stenosis and Bartter


syndrome are associated with hypokalaemia due raised serum
aldosterone.

Aldosterone is elevated in bilateral renal artery stenosis and Bartter


syndrome due to reduced renal perfusion. Aldosterone is high in
primary hyperaldosteronism due to (most commonly) an aldosterone
producing adenoma.

However, serum renin is usually low in primary hyperaldosteronism due


to the resulting hypertension causing excessive renal perfusion, which
results in decreased renin production (negative feedback mechanism).
High renin levels are seen in renal artery stenosis and Bartter syndrome
as a mechanism to improve renal perfusion.

Renal artery stenosis is associated with hypertension and may be


associated with abdominal bruits caused by turbulent flow within the
stenosed arteries.

Bartter syndrome is associated with normotension.

The commonest cause of bilateral renal artery stenosis is


atherosclerosis. However, in this scenario the patient is young and no
cardiovascular risk factors are mentioned. Another cause of bilateral
renal artery stenosis, which should be considered in this patient, is
fibromuscular dysplasia, which can result in a non-atherosclerotic renal
artery stenosis.

CamScanner
DISCUSS { 4 ) improve

Next question >


Renal vascular disease

Renal vascular disease is most commonly due to atherosclerosis (>


95% of patients). It is associated with risk factors such as smoking and
hypertension that cause atheroma elsewhere in the body. It may present
as hypertension, chronic renal failure or ’flash' pulmonary oedema. In
younger patients however fibromuscular dysplasia (FMD) needs to be
considered. FMD is more common in young women and
characteristically has a 'string of beads' appearance on angiography.
Patients respond well to balloon angioplasty

Investigation
• MR angiography is now the investigation of choice
• CT angiography
• conventional renal angiography is less commonly performed used
nowadays, but may still have a role when planning surgery

Next question >

B I B A mM
= " xr H" H ea

Save my notes

Search

Search textbook...

CamScanner -; -^1
£>3
G Q6 P ©
A young male on peritoneal dialysis (PD) presents with abdominal pain
and inflammation around the extension site of his catheter. As well as
this, he is experiencing general malaise and has vomited on two
occasions. Regarding dialysis, he confesses that his standards of
cleanliness 'could be better '.

On examination, there is abdominal tenderness and guarding. The


peritoneal bag contains cloudy effluent.

Analysis of peritoneal fluid shows a white cell count of 430 per mmA 3
with the majority being polymorphonuclear leukocytes.

Culture of the effluent would most likely isolate which of the following
organisms?

Escherichia coli

Pseudomonas

Staphylococcus aureus

Staphylococcus epidermidis

Streptococcus pneumoniae

Submit answer

CamScanner
A 34-year-old woman presents to your GP clinic for ongoing
investigation of her hypertension.

She has recently completed ambulatory blood pressure monitoring


which found an average blood pressure of 152/ 98mmHg. Examination
is unremarkable. pHCG is negative.

She is normally fit and well apart from a history of childhood asthma.
She is teetotal but smokes 5 cigarettes a day. Her father passed away
last year at the age of 62 from a stroke.

Urine dip is normal. You suspect fibromuscular dysplasia.

Which of the following investigation results would be most typical in


fibromuscular dysplasia ?

Echocardiogram showing left outflow obstruction

Serum cholesterol >7

Ultrasound renal tracts showing asymmetric kidneys

MRI abdomen showing solitary adrenal mass

Electrolyte profile showing hypokalaemia

Young female, hypertension and asymmetric kidneys


fibromuscular dysplasia
-
Importance: 69

A patients who is young, female, hypertension and has asymmetric


kidneys with a normal urine dip should prompt consideration of
fibromuscular dysplasia.

Assymmetric kidneys are often first noted on ultrasound; imaging of the

CamScanner
Assymmetric kidneys are often first noted on ultrasound; imaging of the
vessel is done by duplex and most accurately by contrast catheter
angiography of the renal artery.

Examination was unremarkable and there are no symptoms of


cardiovascular disease. Left outflow obstruction would be found in
coarctation of aorta, another secondary cause of hypertension.

Fibromuscular dysplasia is not an atherosclerotic process and it does


not have the same modifiable 'lifestyle' risk factors as renal artery
stenosis, including hypercholesterolaemia .

Conn's syndrome and phaeochromocytoma might present with an


adrenal mass . Conn's syndrome might also present with hypokalaemia .

A f # Discuss Improve

Next question >

Fibromuscular dysplasia

Renal artery stenosis secondary to atherosclerosis accounts for around


90 % of renal vascular disease, with fibromuscular dysplasia being the
most common cause of the remaining 10%.

Epidemiology
• 90% of patients are female

Features
• hypertension
• chronic kidney disease or more acute renal failure e . g . secondary
to ACE-inhibitor initiation
• 'flash' pulmonary oedema

CamScanner
Q3

Each one of the following is a cause of cranial diabetes insipidus,


except:

Pituitary surgery

Lithium

Histiocytosis X

Craniopharyngioma

Post head-injury

Submit answer

Reference ranges v

Score: 100%

1 V
2 >/

CamScanner
Pituitary surgery

Lithium

Histiocytosis X

Craniopharyngioma

Post head-injury

Lithium causes a nephrogenic diabetes insipidus

«
* f # Discuss Improve

Next question >


Diabetes insipidus

Diabetes insipidus (Dl) is a condition characterised by either a


deficiency of antidiuretic hormone, ADH, (cranial Dl) or an insensitivity
to antidiuretic hormone (nephrogenic Dl).

Causes of cranial Dl
• idiopathic
• post head injury
• pituitary surgery
• craniopharyngiomas
• histiocytosis X
• DIDMOAD is the association of cranial Diabetes Insipidus,
Diabetes Mellitus, Optic Atrophy and Deafness (also known as
Wolfram's syndrome)
• haemochromatosis

CamScanner
Causes of nephrogenic Dl
• genetic: the more common form affects the vasopression (ADH)
receptor, the less common form results from a mutation in the
gene that encodes the aquaporin 2 channel
• electrolytes: hypercalcaemia, hypokalaemia
• drugs: demeclocycline, lithium
• tubulo-interstitial disease: obstruction, sickle-cell, pyelonephritis

Features
• polyuria
• polydipsia

Investigation
• high plasma osmolality, low urine osmolality
• a urine osmolality of >700 mOsm/ kg excludes diabetes insipidus
• water deprivation test

Management
• nephrogenic diabetes insipidus: thiazides, low salt / protein diet
- central diabetes insipidus can be treated with desmopressin

Next question >

B / a A •M

•mmm EL - Tr E c-o

Save my notes

Search

CamScanner
Q Q5 P ©
A 62-year -old man is diagnosed with renal cell cancer. Which one of the
following hormones is least likely to be present in excessive levels?

Erythropoietin

Parathyroid hormone

Growth hormone

ACTH

Renin

Submit answer

Reference ranges >/

Score: 75%

1
2 >/

3 X
4 >/

CamScanner
Q5

A 62-year-old man is diagnosed with renal cell cancer. Which one of the
following hormones is least likely to be present in excessive levels?

Erythropoietin

Parathyroid hormone

Growth hormone

ACTH

Renin

# Discuss (3)
* Improve

Next question >


Renal cell cancer

Renal cell cancer is also known as hypernephroma and accounts for


85% of primary renal neoplasms. It arises from proximal renal tubular
epithelium. The most common histological subtype is clear cell (75 to
85 percent of tumours).

Associations*
• more common in middle-aged men
• smoking
• von Hippel-Lindau syndrome
• tuberous sclerosis

CamScanner
Features
• classical triad: haematuria, loin pain, abdominal mass
• pyrexia of unknown origin
• left varicocele (due to occlusion of left testicular vein)
• endocrine effects: may secrete erythropoietin (polycythaemia),
parathyroid hormone (hypercalcaemia), renin, ACTH
• 25% have metastases at presentation
• paraneoplastic hepatic dysfunction syndrome. Also known as
Stauffer syndrome. Typically presents as
cholestasis / hepatosplenomegaly. It is thought to be secondary to
increased levels of IL- 6

Management
• for confined disease a partial or total nephrectomy depending on
the tumour size
• alpha-interferon and interleukin-2 have been used to reduce
tumour size and also treat patients with metatases
• receptor tyrosine kinase inhibitors (e.g. sorafenib, sunitinib) have
been shown to have superior efficacy compared to interferon-
alpha

CamScanner
i passmedicine.com / question / qu a

EM
mage sourced from Wikipedi « i Image used on li ise from PathoPic

Left: normal kidney. Right: 'clear-cell' pattern of renal cell


carcinoma

) Image used on licen rathoPic

' Clear-cell' pattern of renal cell carcinoma - clear cytoplasm,


small nuclei

incidence of renal cell cancer is only slightly increased in patients with


autosomal dominant polycystic kidney disease

CamScanner
o Q8 p o
A 38-year-old woman is referred by the general practitioner as her
recent blood test showed elevated creatinine and urea levels. Other
blood test results are normal. She is oliguric but could produce enough
sample for urine dip. Her urine is negative for red cells, nitrites and
leucocytes. She was recently started on Ramipril for hypertension 3
weeks ago.

She is otherwise well. Her observations are within normal limits. The
renal ultrasound shows two normal-sized kidneys with an unobstructed
urinary system. Duplex ultrasound showed stenotic renal arteries.
Subsequent MR angiography confirmed the diagnosis with the finding
of ’string of beads' appearance of the renal arteries.

What is the most likely cause of this presentation?

Glomerulonephritis

Pyelonephritis

Fibromuscular dysplasia

Nephrotic syndrome

Nephrolithiasis

Submit answer

CamScanner
Fibromuscular dysplasia

Nephrotic syndrome

Nephrolithiasis

Consider fibromuscular dysplasia in young female patients who


develop AKI after the initiation of an ACE inhibitor
Importance: 87

In young female patients who develop AKI after initiation of an ACE


inhibitor, fibromuscular dysplasia should be suspected. In older
persons, suspect atherosclerosis of renal arteries.

Fibromuscular dysplasia describes the proliferation of cells in the walls


of the arteries causing the vessels to bulge or narrow. This most
commonly affects women. These patients are susceptible to AKI after
the initiation of an ACE inhibitor. The classic description is 'string of
beads' appearance.

The normal urine dip results ruled out glomerulonephritis and nephrotic
syndrome as the cause of kidney failure. Glomerulonephritis typically
presents with haematuria with the presence of red cell casts. Nephrotic
syndrome presents with heavy proteinuria, oedema and
hypoalbuminaemia.

Nephrolithiasis typically presents with painful loin-to-groin pain. It does


not cause acute kidney injury unless both kidneys are affected. This
was ruled out by the renal ultrasound.

i
* "t # Discuss (1) Improve

Next question >


CamScanner -;
© Q9 ©
A 35-year-old woman presents with a one week history of progressive
leg swelling. Her past medical history includes type 2 diabetes which is
diet-controlled. On examination, there is bilateral pitting oedema up to
her knees and periorbital oedema. Her observations are heart rate
88 / min, blood pressure 151 / 91 mmHg, oxygen saturations 97%,
temperature 37.1°C, and respiratory rate 14/min. Urine dipstick shows
protein +++. Two days later, she complains of left-sided flank pain and
haematuria.

What complication has occurred?

Haemorrhage into renal cyst

Splenic infarction

Renal vein thrombosis

Haemolytic crisis

Ureteric stone

Submit answer

Reference ranges v

Score: 87.5%

CamScanner
Haemorrhage into renal cyst

Splenic infarction

Renal vein thrombosis

Haemolytic crisis

Ureteric stone

Nephrotic syndrome is associated with a hypercoagulable state


due to loss of antithrombin III via the kidneys
Importance: 77

Nephrotic syndrome is associated with a hypercoagulable state due to


loss of antithrombin III via the kidneys. The most common site of
thrombosis is the renal vein but patients are also at risk of deep vein
thromboses and pulmonary embolisms.

Haemorrhage into a renal cyst may present with flank pain and
haematuria but this history is not suggestive of renal cystic disease.

Splenic infarction can occur in hypercoagulable states but this is less


common than renal vein thrombosis in nephrotic syndrome.
Furthermore, haematuria is not a feature of splenic infarction.

Haemolytic crisis occurs in sickle cell disease, not nephrotic syndrome.

Ureteric stones are not associated with nephrotic syndrome.

Triors .
* i rs r t i

CamScanner -*
Next question ?

Nephrotic syndrome: complications

Complications
• increased risk of infection due to urinary immunoglobulin loss
• increased risk of thromboembolism related to loss of
antithrombin III and plasminogen in the urine. This may result in a
renal vein thrombosis, resulting in a sudden deterioration in renal
function
• hyperlipidaemia
• hypocalcaemia (vitamin D and binding protein lost in urine)
• acute renal failure

Next question >


E I S A =' xr B" H c-D

Save my notes

Search

Search textbook...

Q Google search on "Nephrotic syndrome: complications "

+ Suggest link

*m I
CamScanner
G Q1

A 45-year -old woman with type 1 diabetes mellitus is reviewed in the


diabetes clinic. Three months ago her blood tests were as followed:

K+ 4.5 mmol/ l

Creatinine 116 pmol/l

eGFR 47 ml/ min

At the time she was started on lisinopril to treat both the hypertension
and act as a renoprotective agent. Lisinopril had been titrated up to
treatment dose. Her current bloods are as follows:

K+ 4.9 mmol/l

Creatinine 123 pmol/l

eGFR 44 ml / min

Of the following options, what is the most appropriate course of action?

Stop lisinopril and arrange investigations to exclude renal artery


stenosis

Switch to a angiotensin 2 receptor blocker

Switch to a different ACE inhibitor

No action

CamScanner
Switch to a different ACE inhibitor

No action

Reduce dose of lisinopril

The small change in both the creatinine and eGFR are acceptable and
below the threshold where ACE inhibitors should be stopped

4 f # Discuss (2) Improve

Next question >


Chronic kidney disease: hypertension

The majority of patients with chronic kidney disease (CKD) will require
more than two drugs to treat hypertension. ACE inhibitors are first line
and are particularly helpful in proteinuric renal disease (e.g. diabetic
nephropathy). As these drugs tend to reduce filtration pressure a small
fall in glomerular filtration pressure (GFR) and rise in creatinine can be
expected. NICE suggest that a decrease in eGFR of up to 25% or a rise
in creatinine of up to 30% is acceptable, although any rise should
prompt careful monitoring and exclusion of other causes (e. g. NSAIDs).
A rise greater than this may indicate underlying renovascular disease.

Furosemide is useful as a anti-hypertensive in patients with CKD,


particularly when the GFR falls to below 45 ml / min*. It has the added
benefit of lowering serum potassium. High doses are usually required. If
the patient becomes at risk of dehydration (e.g. Gastroenteritis) then
consideration should be given to temporarily stopping the drug

*the NKF K / DOQI guidelines suggest a lower cut-off of less than 30


ml/ min

Next question >


CamScanner
© Q2 \C3 ©
A 40-year -old man presents to the renal out -patient clinic after being
referred by his GP following an incidental finding of microscopic
haematuria at an insurance medical. He has never noted frank
haematuria or renal angle pain before, and has otherwise been well. He
denies any recent upper respiratory tract infections. There is no family
history of renal failure or renal disease, although his brother and father
have previously seen specialists because of 'blood in the urine',
although both have since been discharged. On examination his blood
pressure is 124/ 74 mmHg and his creatinine is in the normal range. A
renal USS performed by his GP at the time of referral is normal. What is
the most likely diagnosis?

Thin basement membrane disease

IgA nephropathy

Renal lithiasis

Acute interstitial nephritis

Post streptococcal glomerulonephritis

Submit answer

Reference ranges v

CamScanner
although both have since been discharged. On examination his blood
pressure is 124/ 74 mmHg and his creatinine is in the normal range. A
renal USS performed by his GP at the time of referral is normal. What is
the most likely diagnosis?

Thin basement membrane disease

IgA nephropathy

Renal lithiasis

Acute interstitial nephritis

Post streptococcal glomerulonephritis

A t # Discuss (4) Improve

Next question >


Thin basement membrane disease

An inherited disorder of type IV collagen that causes thinning of the


basement membrane. It may affect up to 5% of the population and
about 30% patients report a family history of haematuria. Diagnosis is
usually based on the history of persistent haematuria, normal kidney
function and family history of haematuria without kidney failure. It is
generally a benign disorder and biopsy is rarely indicated.

Next question >

B / B A m
1“
=' T \w SS "” E c-o

CamScanner
Q1 pj

Which one of the following may be useful in the prevention of oxalate


renal stones?

Ferrous sulphate

Thiazide diuretics

Lithium

Pyridoxine

Allopurinol

Submit answer

Reference ranges v

Score: 0%

CamScanner
Lithium

Pyridoxine

Allopurinol

at t # Discuss (2) Improve

Next question >


Renal stones: management

The British Association of Urological Surgeons (BAUS) published


guidelines in 2018 on the management of acute ureteric / renal colic.

Initial management of renal colic

Medication
• the BAUS recommend an NSAID as the analgesia of choice for
renal colic
• whilst diclofenac has been traditionally used the increased risk of
cardiovascular events with certain NSAIDs (e. g. diclofenac,
ibuprofen) should be considered when prescribing
• the CKS guidelines suggest for patients who require admission:
' Administer a parenteral analgesic (such as intramuscular
diclofenac ) for rapid relief of severe pain'
• BAUS no longer endorse the use of alpha- adrenergic blockers to
aid ureteric stone passage routinely. They do however
acknowledge a recently published meta-analysis advocates the
use of a-blockers for patients amenable to conservative
management, with greatest benefit amongst those with larger
stones

CamScanner
Initial investigations
• urine dipstick and culture
• serum creatinine and electrolytes: check renal function
• FBC / CRP: look for associated infection
• calcium / urate: look for underlying causes
• also: clotting if percutaneous intervention planned and blood
cultures if pyrexial or other signs of sepsis

Imaging
• BAUS now recommend that non-contrast CT KUB should be
performed on all patients, within 14 hours of admission
• if a patient has a fever, a solitary kidney or when the diagnosis is
uncertain an immediate CT KUB should be performed. In the case
of an uncertain diagnosis, this is to exclude other diagnoses such
as ruptured abdominal aortic aneurysm
• CT KUB has a sensitivity of 97% for ureteric stones and a
specificity of 95%
• ultrasound still has a role but given the wider availability of CT
now and greater accurary it is no longer recommend first -line. The
sensitivity of ultrasound for stones is around 45% and specificity
is around 90%

Management of renal stones

Stones < 5 mm will usually pass spontaneously. Lithotripsy and


nephrolithotomy may be for severe cases.

Most renal stones measuring less than 5mm in maximum diameter will
typically pass within 4 weeks of symptom onset. More intensive and
urgent treatment is indicated in the presence of ureteric obstruction,
renal developmental abnormality such as horseshoe kidney and
previous renal transplant. Ureteric obstruction due to stones together
with infection is a surgical emergency and the system must be
decompressed. Options include nephrostomy tube placement, insertion
of ureteric catheters and ureteric stent placement.
CamScanner
• CT KUB has a sensitivity of 97% for ureteric stones and a specificity of 95%
• ultrasound still has a role but given the wider availability of CT now and
greater accurary it is no longer recommend first-line. The sensitivity of
ultrasound for stones is around 45% and specificity is around 90%

Management of renal stones

Stones < 5 mm will usually pass spontaneously. Lithotripsy and nephrolithotomy


may be for severe cases.

Most renal stones measuring less than 5mm in maximum diameter will typically
pass within 4 weeks of symptom onset . More intensive and urgent treatment is
indicated in the presence of ureteric obstruction, renal developmental
abnormality such as horseshoe kidney and previous renal transplant. Ureteric
obstruction due to stones together with infection is a surgical emergency and
the system must be decompressed. Options include nephrostomy tube
placement, insertion of ureteric catheters and ureteric stent placement.

In the non-emergency setting, the preferred options for treatment of stone


disease include extra corporeal shock wave lithotripsy, percutaneous
nephrolithotomy, ureteroscopy, open surgery remains an option for selected
cases. However, minimally invasive options are the most popular first-line
treatment.

Shockwave lithotripsy
• A shock wave is generated external to the patient, internally cavitation
bubbles and mechanical stress lead to stone fragmentation. The passage
of shock waves can result in the development of solid organ injury.
Fragmentation of larger stones may result in the development of ureteric
obstruction. The procedure is uncomfortable for patients and analgesia is
required during the procedure and afterwards.

Ureteroscopy
• A ureteroscope is passed retrograde through the ureter and into the renal
pelvis. It is indicated in individuals (e.g. pregnant females) where
CamScanner
• CT KUB has a sensitivity of 97% for ureteric stones and a specificity of 95%
• ultrasound still has a role but given the wider availability of CT now and
greater accurary it is no longer recommend first-line. The sensitivity of
ultrasound for stones is around 45% and specificity is around 90%

Management of renal stones

Stones < 5 mm will usually pass spontaneously. Lithotripsy and nephrolithotomy


may be for severe cases.

Most renal stones measuring less than 5mm in maximum diameter will typically
pass within 4 weeks of symptom onset. More intensive and urgent treatment is
indicated in the presence of ureteric obstruction, renal developmental
abnormality such as horseshoe kidney and previous renal transplant. Ureteric
obstruction due to stones together with infection is a surgical emergency and
the system must be decompressed. Options include nephrostomy tube
placement, insertion of ureteric catheters and ureteric stent placement .

In the non-emergency setting, the preferred options for treatment of stone


disease include extra corporeal shock wave lithotripsy, percutaneous
nephrolithotomy, ureteroscopy, open surgery remains an option for selected
cases. However, minimally invasive options are the most popular first -line
treatment.

Shockwave lithotripsy
• A shock wave is generated external to the patient, internally cavitation
bubbles and mechanical stress lead to stone fragmentation. The passage
of shock waves can result in the development of solid organ injury.
Fragmentation of larger stones may result in the development of ureteric
obstruction. The procedure is uncomfortable for patients and analgesia is
required during the procedure and afterwards.

Ureteroscopy
• A ureteroscope is passed retrograde through the ureter and into the renal
pelvis. It is indicated in individuals (e. g. pregnant females) where
CamScanner
Ureteroscopy
• A ureteroscope is passed retrograde through the ureter and into the renal
pelvis. It is indicated in individuals (e.g. pregnant females) where
lithotripsy is contraindicated and in complex stone disease. In most cases
a stent is left in situ for 4 weeks after the procedure.

Percutaneous nephrolithotomy
• In this procedure, access is gained to the renal collecting system. Once
access is achieved, intra corporeal lithotripsy or stone fragmentation is
performed and stone fragments removed.

Therapeutic selection

Disease Option

Stone burden of less than 2cm in aggregate Lithotripsy

Stone burden of less than 2cm in pregnant Ureteroscopy


females

Complex renal calculi and staghorn calculi Percutaneous


nephrolithotomy

Ureteric calculi less than 5mm Manage expectantly

Prevention of renal stones

Calcium stones may be due to hypercalciuria, which is found in up to 5-10% of


the general population.
• high fluid intake
• low animal protein, low salt diet (a low calcium diet has not been shown to
be superior to a normocalcaemic diet)
• thiazides diuretics (increase distal tubular calcium resorption)
CamScanner
nephrolithotomy

Ureteric calculi less than 5mm Manage expectantly

Prevention of renal stones

Calcium stones may be due to hypercalciuria, which is found in up to 5-10% of


the general population.
• high fluid intake
• low animal protein, low salt diet (a low calcium diet has not been shown to
be superior to a normocalcaemic diet)
• thiazides diuretics (increase distal tubular calcium resorption)

Oxalate stones
• cholestyramine reduces urinary oxalate secretion
• pyridoxine reduces urinary oxalate secretion

Uric acid stones


• allopurinol
• urinary alkalinization e.g. oral bicarbonate

B / a A
m
H^ S c-o X

Save my notes

CamScanner
Q2 pj

Each one of the following is a cause of nephrogenic diabetes insipidus,


except:

Hypercalcaemia

Demeclocycline

Histiocytosis X

Lithium

Hypokalaemia

Submit answer

Reference ranges v

Score: 100%

1 V
2

CamScanner
except:

Hypercalcaemia

Demeclocycline

Histiocytosis X

Lithium

Hypokalaemia

a
* -* # Discuss (2) Improve

Next question >

Diabetes insipidus

Diabetes insipidus (Dl) is a condition characterised by either a


deficiency of antidiuretic hormone, ADH, (cranial Dl) or an insensitivity
to antidiuretic hormone (nephrogenic Dl).

Causes of cranial Dl
• idiopathic
• post head injury
• pituitary surgery
• craniopharyngiomas
• histiocytosis X
• DIDMOAD is the association of cranial Diabetes Insipidus,
Diabetes Mellitus, Optic Atrophy and Deafness (also known as
Wolfram's syndrome)
• haemochromatosis
^-
CamScanner -! 1 9 =
• haemochromatosis

Causes of nephrogenic Dl
• genetic: the more common form affects the vasopression (ADH)
receptor, the less common form results from a mutation in the
gene that encodes the aquaporin 2 channel
• electrolytes: hypercalcaemia, hypokalaemia
• drugs: demeclocycline, lithium
• tubulo-interstitial disease: obstruction, sickle-cell, pyelonephritis

Features
• polyuria
• polydipsia

Investigation
• high plasma osmolality, low urine osmolality
• a urine osmolality of >700 mOsm/kg excludes diabetes insipidus
• water deprivation test

Management
• nephrogenic diabetes insipidus: thiazides, low salt / protein diet
- central diabetes insipidus can be treated with desmopressin

Next question >


B / A • =' H c-D
Hi

B m Tr

Save my notes

CamScanner
Q2 pj

Which of the following types of renal stones are said to have a semi-
opaque appearance on x -ray?

Calcium oxalate

Cystine stones

Urate stones

Xanthine stones

Triple phosphate stones

Submit answer

Reference ranges v

Score: 100%

1 V
2

CamScanner
Q2 p

Which of the following types of renal stones are said to have a semi-
opaque appearance on x-ray?

Calcium oxalate

Cystine stones

Urate stones

Xanthine stones

Triple phosphate stones

Renal stones on x-ray


• cystine stones: semi-opaque
• urate + xanthine stones: radio-lucent
Important for me Less important

BA “f # Discuss Improve

Next question >


Renal stones: imaging

CamScanner
The table below summarises the appearance of different types of renal
stone on x-ray

Radiograph
Type Frequency appearance

Calcium oxalate 40% Opaque

Mixed calcium 25% Opaque


oxalate/ phosphate stones

Triple phosphate stones* 10% Opaque

Calcium phosphate 10% Opaque

Urate stones 5-10% Radio-lucent

Cystine stones 1% Semi-opaque, 'ground-


glass ' appearance

Xanthine stones <1% Radio-lucent

*stag-horn calculi involve the renal pelvis and extend into at least 2
calyces. They develop in alkaline urine and are composed of struvite
(ammonium magnesium phosphate, triple phosphate). Ureaplasma
urealyticum and Proteus infections predispose to their formation

Next question >

CamScanner
© Q3

A 68-year-old man presents to general practice with a 3 month history


of progressive dyspnoea, 6kg of unintentional weight loss and a
productive cough of white sputum. Upon further questioning, he admits
bone and joint pain with some constipation and polyuria.

His medical history includes: hypertension, benign prostatic


hyperplasia. His hypertension was well controlled until 3 months ago.

On examination there is digital clubbing, coarse bibasal crackles and


mild epigastric pain, with suprapubic dullness. There is a left-sided
varicocele that enlarges on standing.

His observations were as follows:


Blood pressure - 180 /109 mmHg, heart rate - 95 beats per minute,
temperature - 37.1 °C, oxygen saturation - 95% on air, respiratory rate -
22 breaths per min.

He was referred to the emergency department, where blood tests


revealed:

Hb 94 g/ L Male: (135-180)

Platelets 231 * 109/ L (150 - 400)

WBC 14.2 * 109/ L (4.0 - 11.0)

Calcium 2.9 mmol/ L (2.1-2.6)

Urinalysis reveals: blood 4+, protein 2+.

What is the most likely diagnosis?

Mesothelioma

- ^ ..
CamScanner l4$ b £> ,^11
HIS ouservauoris were as mnows :
Blood pressure - 180 /109 mmHg, heart rate - 95 beats per minute,
temperature - 37.1 °C, oxygen saturation - 95% on air, respiratory rate -
22 breaths per min.

He was referred to the emergency department, where blood tests


revealed:

Hb 94 g/ L Male: (135-180)

Platelets 231 * 109/ L (150 - 400)

WBC 14.2 * 109 / L (4.0 - 11.0)

Calcium 2.9 mmol/ L (2.1-2.6)

Urinalysis reveals: blood 4+, protein 2+.

What is the most likely diagnosis?

Mesothelioma

Prostatic cancer

Small cell lung cancer

Renal cell carcinoma

Phaeochromocytoma

Submit answer

Reference ranges v

Score: 100%
CamScanner ^£>.,^11
Renal cell carcinoma

Phaeochromocytoma

Renal cell carcinoma can metastasise to the lungs, and remains


an important differential in the setting of hypertension,
hypercalcaemia and haematuria
Important for me Less important

Renal cell carcinoma presents with the classic triad only 10% of the
time, and so requires recognition of the other features that are specific
to the pathophysiology of the disease. Other paraneoplastic syndromes
of renal cell carcinoma include polycythaemia and amyloidosis.

Chest X-ray in this patient would show pulmonary cannonball


metastases with mediastinal lymphadenopathy.

This patient 's epigastric pain is likely due to peptic ulcer disease
secondary to hypercalcaemia, his suprapubic dullness likely due to
chronic urinary retention secondary to his benign prostatic hyperplasia.

Option 1. Mesothelioma would explain the respiratory symptoms,


weight loss and clubbing. However it does not explain the
hypercalcaemia nor the varicocele.

Option 2. Prostatic cancer often causes bony metastasis, with potential


subsequent hypercalcaemia, alongside with haematuria. While it can
metastasise to the lungs, it would not explain varicocele nor refractory
hypertension.

Option 3. Small cell lung cancer commonly causes paraneoplastic


syndromes, including Cushing's syndrome, SIADH, Lambert Eaton, and
cerebellar degeneration. However, squamous cell cancer is the cause of
hypercalcaemia of malignancy, and it does not explain the varicocele
nor the haematuria.
CamScanner
Renal cell cancer

Renal cell cancer is also known as hypernephroma and accounts for


85% of primary renal neoplasms. It arises from proximal renal tubular
epithelium. The most common histological subtype is clear cell (75 to
85 percent of tumours).

Associations*
• more common in middle- aged men
• smoking
• von Hippel-Lindau syndrome
• tuberous sclerosis

Features
• classical triad: haematuria, loin pain, abdominal mass
• pyrexia of unknown origin
• left varicocele (due to occlusion of left testicular vein)
• endocrine effects: may secrete erythropoietin (polycythaemia),
parathyroid hormone (hypercalcaemia), renin, ACTH
• 25% have metastases at presentation
• paraneoplastic hepatic dysfunction syndrome. Also known as
Stauffer syndrome. Typically presents as
cholestasis/ hepatosplenomegaly. It is thought to be secondary to
increased levels of IL-6

Management
• for confined disease a partial or total nephrectomy depending on
the tumour size
• alpha-interferon and interleukin-2 have been used to reduce
tumour size and also treat patients with metatases
• receptor tyrosine kinase inhibitors (e.g. sorafenib, sunitinib) have
been shown to have superior efficacy compared to interferon-
alpha

CamScanner
A 60-year- old woman presents to her general practice with signs of
thirst and tiredness. On further questioning, she also complains of
polyuria and nocturia. As a result a 24-hour urine is collected, which
returns showing a urine osmolality of 189 mOsm/ kg ( 500-800
mOsm/ kg).

The general practitioner suspects a diagnosis of diabetes insipidus.

Which of the following is a recognised cause of nephrogenic diabetes


insipidus?

Hyperkalaemia

Hypercalcaemia

Histiocytosis X

Hypermagnesemia

Hypernatraemia

Submit answer

Reference ranges >/

CamScanner
© Q8 PJ ©
A 60-year -old woman presents to her general practice with signs of
thirst and tiredness. On further questioning, she also complains of
polyuria and nocturia. As a result a 24-hour urine is collected, which
returns showing a urine osmolality of 189 mOsm/ kg ( 500-800
mOsm/kg).

The general practitioner suspects a diagnosis of diabetes insipidus.

Which of the following is a recognised cause of nephrogenic diabetes


insipidus?

Hyperkalaemia

Hypercalcaemia

Histiocytosis X

Hypermagnesemia

Hypernatraemia

The polyuria and polydipsia, in combination with a dilute urine suggest


diabetes insipidus. It is hypokalaemia, not hyperkalaemia, that causes
nephrogenic diabetes insipidus whereas Histiocytosis X causes cranial
diabetes insipidus.

a6 Bt # Discuss (3) Improve

CamScanner
© Q9 p ©
A 70-year-old man has been admitted with abdominal pain. The
surgeons wish to perform a contrast -enhanced CT but are concerned
because he has chronic kidney disease stage 3. His latest renal
function is shown below:

Na + 142 mmol/l

K+ 4.6 mmol /l

Urea 8.1 mmol/l

Creatinine 130 pmol/l

Which one of the following is the most important step in reducing the
risk of contrast-induced nephropathy?

Oral sodium bicarbonate pre- and post -procedure

Oral N-acetylcysteine pre- and post-procedure

Intravenous 0.9 % sodium chloride pre- and post-procedure

Intravenous furosemide pre-procedure

Intravenous mannitol post -procedure

Submit answer

Reference ranges v

CamScanner
i4r f # Discuss (1) Improve

Next question >


Nephrotoxicity due to contrast media

Contrast media nephrotoxicity may be defined as a 25% increase in


creatinine occurring within 3 days of the intravascular administration of
contrast media.

Risk factors include


• known renal impairment (especially diabetic nephropathy)
• age > 70 years
• dehydration
• cardiac failure
• the use of nephrotoxic drugs such as NSAIDs

Contrast-induced nephropathy occurs 2 - 5 days after administration.

Prevention
• the evidence base currently supports the use of intravenous 0.9%
sodium chloride at a rate of 1 mL/ kg / hour for 12 hours pre- and
post - procedure. There is also evidence to support the use of
isotonic sodium bicarbonate
• N-acetylcysteine has been given in the past but recent evidence
suggests it is not effective*

* Outcomes after Angiography with Sodium Bicarbonate and


Acetylcysteine. N Engl J Med. 2018;378(7):603

Next question >


R I A TTT Effl ’*’ r-~i
CamScanner
o Q 13 ©
A 47-year-old woman presents with loin pain and haematuria. Urine
dipstick demonstrates:

Blood ++++

Nitrites POS

Leucocytes +++

Protein ++

Urine culture shows a Proteus infection. An x -ray demonstrates a stag-


horn calculus in the left renal pelvis. What is the most likely
composition of the renal stone?

Xanthine

Calcium oxalate

Struvite

Cystine

O Urate

Submit answer

Reference ranges v

CamScanner
o Q13 >/ p ©
A 47-year-old woman presents with loin pain and haematuria. Urine
dipstick demonstrates:

Blood ++++

Nitrites POS

Leucocytes +++

Protein ++

Urine culture shows a Proteus infection. An x -ray demonstrates a stag-


horn calculus in the left renal pelvis. What is the most likely
composition of the renal stone?

Xanthine

Calcium oxalate

Struvite

Cystine

Urate

Stag-horn calculi are composed of struvite and form in alkaline


urine (ammonia producing bacteria therefore predispose)
Important for me Less important

BA
^ Discuss (1) Improve
CamScanner
© Q3 p ©
A 50 year-old man with dialysis dependent chronic kidney disease is
awaiting renal transplant. He complains of fatigue. On examination you
note heart rate 95 beats per minute, soft ejection systolic murmur that
doesn't radiate and pallor. There were no other abnormal features.

What is the most likely cause of his fatigue?

Heart failure

Endocarditis

Uraemic encephalopathy

Anaemia

Hyperkalaemia

Submit answer

Reference ranges v

Score: 50%

2 X
3

CamScanner
© Q3 v
' P3 ©
A 50 year -old man with dialysis dependent chronic kidney disease is
awaiting renal transplant. He complains of fatigue. On examination you
note heart rate 95 beats per minute, soft ejection systolic murmur that
doesn't radiate and pallor. There were no other abnormal features.

What is the most likely cause of his fatigue?

Heart failure

Endocarditis

Uraemic encephalopathy

Anaemia

Hyperkalaemia

Anaemia is extremely common in chronic kidney disease. It is often


caused by iron deficiency or erythropoietin deficiency. The man in this
case has a few signs and symptoms of anaemia - tachycardia, fatigue,
pallor and an aortic flow murmur.

>4 ? # Discuss (4) Improve

Next question >

CamScanner
Next question >

Chronic kidney disease: anaemia

Patients with chronic kidney disease (CKD) may develop anaemia due
to a variety of factors, the most significant of which is reduced
erythropoietin levels. This is usually a normochromic normocytic
anaemia and becomes apparent when the GFR is less than 35 ml / min
(other causes of anaemia should be considered if the GFR is > 60
ml/ min) . Anaemia in CKD predisposes to the development of left
ventricular hypertrophy - associated with a three fold increase in
mortality in renal patients

Causes of anaemia in renal failure


• reduced erythropoietin levels - the most significant factor
• reduced erythropoiesis due to toxic effects of uraemia on bone
marrow
• reduced absorption of iron
• anorexia /nausea due to uraemia
• reduced red cell survival (especially in haemodialysis)
• blood loss due to capillary fragility and poor platelet function
• stress ulceration leading to chronic blood loss

Management
• the 2011 NICE guidelines suggest a target haemoglobin of 10 - 12
g/ dl
• determination and optimisation of iron status should be carried
out prior to the administration of erythropoiesis-stimulating
agents (ESA). Many patients, especially those on haemodialysis,
will require IV iron
• ESAs such as erythropoietin and darbepoetin should be used in
those ' who are likely to benefit in terms of quality of life and
physical function'

Next question >


CamScanner
© Q4 p ©
A 29-year-old man has his renal function checked. The eGFR is
calculated to be 54 ml/ min. Which one of the following factors is most
likely to explain this unexpectedly low result ?

Drinking a large amount of milk

Being dehydrated when the blood sample was taken

Being very tall

Excessive alcohol intake

Large muscle mass secondary to body building

Submit answer

Reference ranges v

Score: 66.7%

1
2 X
3
4

CamScanner
A 29- year- old man has his renal function checked . The eGFR is
calculated to be 54 ml / min . Which one of the following factors is most
likely to explain this unexpectedly low result ?

Drinking a large amount of milk

Being dehydrated when the blood sample was taken

Being very tall

Excessive alcohol intake

Large muscle mass secondary to body building

The eGFR is often inaccurate in people with extremes of muscle mass.


Body builders often have an inappropriately low eGFR .

A Bf # Discuss (3) Improve

Next question >

Chronic kidney disease: eGFR and


classification

Serum creatinine may not provide an accurate estimate of renal


function due to differences in muscle. For this reason formulas were
develop to help estimate the glomerular filtration rate (estimated GFR or
eGFR ) . The most commonly used formula is the Modification of Diet in
Renal Disease (MDRD) equation, which uses the following variables:
• serum creatinine
• age
• gender
• ethnicity

CamScanner
Factors which may affect the result
• pregnancy
• muscle mass (e.g. amputees, body-builders)
• eating red meat 12 hours prior to the sample being taken

CKD may be classified according to GFR:

CKD
stage GFR range

1 Greater than 90 ml/ min, with some sign of kidney damage


on other tests (if all the kidney tests* are normal, there is
no CKD)

2 60-90 ml/ min with some sign of kidney damage (if kidney
tests* are normal, there is no CKD)

3a 45- 59 ml/ min, a moderate reduction in kidney function

3b 30-44 ml/ min, a moderate reduction in kidney function

4 15-29 ml/ min, a severe reduction in kidney function

5 Less than 15 ml/ min, established kidney failure - dialysis


or a kidney transplant may be needed

*i.e. normal U&Es and no proteinuria

Next question >

B / a A ""
"B
BB
BB
=~ Tlw SS '*' E c-o

CamScanner
G Q9 P O
A 56-year-old man who suffers from biventricular cardiac failure comes
to the clinic for review. His symptoms are currently well controlled and
he is taking Ramipril 1 Omg, spironolactone 25mg, bisoprolol 1Omg, and
furosemide 40mg. His main complaint is of painful gynaecomastia that
he says has developed over the past 6 months. Physical exam reveals a
blood pressure of 125/ 80 mmHg, and no residual signs of cardiac
failure. Renal function is unchanged from 6 months earlier, with a stable
creatinine at 125 pmol /l.

Which of the following is the most appropriate next step?

Check is prolactin level

Change the spironolactone to eplerenone

Change the furosemide to indapamide

Stop the bisoprolol

Check his testosterone level

Submit answer

Reference ranges

Score: 87.5%

CamScanner
Which of the following is the most appropriate next step?

Check is prolactin level

Change the spironolactone to eplerenone

Change the furosemide to indapamide

Stop the bisoprolol

Check his testosterone level

Eplerenone can be used in patients with troublesome


gynaecomastia on spironolactone
Important for me Less important

This man has stable heart failure, his medication should therefore
remain unaltered if possible. Spironolactone does however interfere
with binding of testosterone to androgen receptors, increase metabolic
clearance of testosterone, and results in increased metabolism of
androgens to estradiol. This is the likely cause of his gynaecomastia.
Eplerenone is an alternative aldosterone antagonist and is associated
with up to 20 times less symptom burden with respect to
gynaecomastia. It is therefore the most appropriate intervention here.

https:// www.medicines.org.uk / emc / medicine/ 29837

At this stage there is little to be gained by checking the patient 's


testosterone level until the spironolactone is discontinued. Although
bisoprolol may be associated with erectile dysfunction, this isn't
mediated by anti-androgen activity, and it is not linked to the
development of gynaecomastia. Changing the furosemide to a thiazide
like diuretic may worsen symptoms of heart failure. High levels of
prolactin are associated with galactorrhoea, not with gynaecomastia.
There is little value therefore in checking prolactin levels here.
CamScanner
<r
i passmedicine.com / question / qu
^ i O
^ Cl

Spironolactone

Spironolactone is an aldosterone antagonist which acts in the cortical


collecting duct.

Indications
• ascites: patients with cirrhosis develop a secondary
hyperaldosteronism. Relatively large doses such as 100 or 200mg
are often used
• hypertension: used in some patients as a NICE 'step 4' treatment
• heart failure (see RALES study below)
• nephrotic syndrome
• Conn's syndrome

Adverse effects
• hyperkalaemia
• gynaecomastia: less common with eplerenone

RALES
• NYHA III + IV, patients already taking ACE inhibitor
• low dose spironolactone reduces all cause mortality

Next question >


B / a A m HI =' Tr SS ,r E CH>

Save my notes

CamScanner
Q Q11 |cn ©
A 69-year -old woman is admitted with a third episode of urinary tract
infection over the past 12 months, proven on urine culture to be due to
Proteus mirabilis. She also has right loin pain. She has a past history of
hypertension and a previous creatinine tested one month earlier was
elevated at 145 micromol / l. A plain x -ray to display kidneys ureters and
bladder and an ultrasound of the urinary tract reveal a right staghorn
calculus.

What is the stone likely to be composed of ?

Calcium phosphate

Urate

Magnesium ammonium phosphate

Cystine

Oxalate

Submit answer

Reference ranges >

CamScanner
Urate

Magnesium ammonium phosphate

Cystine

Oxalate

Magnesium ammonium phosphate, (also known as struvite), kidney


stones are formed by urea- splitting bacteria, of which Proteus is an
example. Magnesium ammonium phosphate stones can form very large
calculi known as staghorn calculi which may significantly impact on
renal function due to chronic obstruction and failure to clear urinary
tract infection adequately. Treatment of the underlying urinary tract
infection, coupled with a urology consult to assess the need for removal
of any larger stones are the cornerstone of management .

Calcium phosphate renal stones are associated with


hyperparathyroidism, and urate stones are associated with gout .
Cystine stones are seen in patients with inherited cystinuria, and oxalate
stones are primarily associated with short bowel syndrome.

f # Discuss Improve

Next question >

Renal stones: management

The British Association of Urological Surgeons ( BAUS) published


guidelines in 2018 on the management of acute ureteric / renal colic.
CamScanner
Renal stones: management

The British Association of Urological Surgeons (BAUS) published


guidelines in 2018 on the management of acute ureteric / renal colic.

Initial management of renal colic

Medication
• the BAUS recommend an NSAID as the analgesia of choice for
renal colic
• whilst diclofenac has been traditionally used the increased risk of
cardiovascular events with certain NSAIDs (e.g. diclofenac,
ibuprofen) should be considered when prescribing
• the CKS guidelines suggest for patients who require admission:
' Administer a parenteral analgesic (such as intramuscular

diclofenac ) for rapid relief of severe pain'


• BAUS no longer endorse the use of alpha-adrenergic blockers to
aid ureteric stone passage routinely. They do however
acknowledge a recently published meta-analysis advocates the
use of a-blockers for patients amenable to conservative
management, with greatest benefit amongst those with larger
stones

Initial investigations
• urine dipstick and culture
• serum creatinine and electrolytes: check renal function
• FBC / CRP: look for associated infection
• calcium/urate: look for underlying causes
• also: clotting if percutaneous intervention planned and blood
cultures if pyrexial or other signs of sepsis

Imaging
• BAUS now recommend that non-contrast CT KUB should be
performed on all patients, within 14 hours of admission
• if a patient has a fever, a solitary kidney or when the diagnosis is
nr»nr + oin nm immnrlio+n PT \/ \ IQ rhm ilrl kn nnrfnrmnrl In +hn nnnn

CamScanner
Renal stones: management

The British Association of Urological Surgeons (BAUS) published


guidelines in 2018 on the management of acute ureteric / renal colic.

Initial management of renal colic

Medication
• the BAUS recommend an NSAID as the analgesia of choice for
renal colic
• whilst diclofenac has been traditionally used the increased risk of
cardiovascular events with certain NSAIDs (e.g. diclofenac,
ibuprofen) should be considered when prescribing
• the CKS guidelines suggest for patients who require admission:
' Administer a parenteral analgesic (such as intramuscular

diclofenac ) for rapid relief of severe pain'


• BAUS no longer endorse the use of alpha-adrenergic blockers to
aid ureteric stone passage routinely. They do however
acknowledge a recently published meta-analysis advocates the
use of a-blockers for patients amenable to conservative
management, with greatest benefit amongst those with larger
stones

Initial investigations
• urine dipstick and culture
• serum creatinine and electrolytes: check renal function
• FBC / CRP: look for associated infection
• calcium/urate: look for underlying causes
• also: clotting if percutaneous intervention planned and blood
cultures if pyrexial or other signs of sepsis

Imaging
• BAUS now recommend that non-contrast CT KUB should be
performed on all patients, within 14 hours of admission
• if a patient has a fever, a solitary kidney or when the diagnosis is
nr»nr + oin nm immnrlio+n PT \/ \ IQ rhm ilrl kn nnrfnrmnrl In +hn nnnn

CamScanner
LUIIUICJ ii pyicAiai ui umei Jiyiu ui scp -su

Imaging
• BAUS now recommend that non-contrast CT KUB should be
performed on all patients, within 14 hours of admission
• if a patient has a fever, a solitary kidney or when the diagnosis is
uncertain an immediate CT KUB should be performed. In the case
of an uncertain diagnosis, this is to exclude other diagnoses such
as ruptured abdominal aortic aneurysm
• CT KUB has a sensitivity of 97% for ureteric stones and a
specificity of 95%
• ultrasound still has a role but given the wider availability of CT
now and greater accurary it is no longer recommend first-line. The
sensitivity of ultrasound for stones is around 45% and specificity
is around 90%

Management of renal stones

Stones < 5 mm will usually pass spontaneously. Lithotripsy and


nephrolithotomy may be for severe cases.

Most renal stones measuring less than 5mm in maximum diameter will
typically pass within 4 weeks of symptom onset. More intensive and
urgent treatment is indicated in the presence of ureteric obstruction,
renal developmental abnormality such as horseshoe kidney and
previous renal transplant. Ureteric obstruction due to stones together
with infection is a surgical emergency and the system must be
decompressed. Options include nephrostomy tube placement, insertion
of ureteric catheters and ureteric stent placement.

In the non-emergency setting, the preferred options for treatment of


stone disease include extra corporeal shock wave lithotripsy,
percutaneous nephrolithotomy, ureteroscopy, open surgery remains an
option for selected cases. However, minimally invasive options are the
most popular first-line treatment.

CamScanner
Shockwave lithotripsy
• A shock wave is generated external to the patient, internally
cavitation bubbles and mechanical stress lead to stone
fragmentation. The passage of shock waves can result in the
development of solid organ injury. Fragmentation of larger stones
may result in the development of ureteric obstruction. The
procedure is uncomfortable for patients and analgesia is required
during the procedure and afterwards.

Ureteroscopy
• A ureteroscope is passed retrograde through the ureter and into
the renal pelvis. It is indicated in individuals (e.g. pregnant
females) where lithotripsy is contraindicated and in complex
stone disease. In most cases a stent is left in situ for 4 weeks
after the procedure.

Percutaneous nephrolithotomy
• In this procedure, access is gained to the renal collecting system.
Once access is achieved, intra corporeal lithotripsy or stone
fragmentation is performed and stone fragments removed.

Therapeutic selection

Disease Option

Stone burden of less than 2cm in Lithotripsy


aggregate

Stone burden of less than 2cm in Ureteroscopy


pregnant females

Complex renal calculi and staghorn Percutaneous


calculi nephrolithotomy

Ureteric calculi less than 5mm Manage expectantly


CamScanner
Therapeutic selection

Disease Option

Stone burden of less than 2cm in Lithotripsy


aggregate

Stone burden of less than 2cm in Ureteroscopy


pregnant females

Complex renal calculi and staghorn Percutaneous


calculi nephrolithotomy

Ureteric calculi less than 5mm Manage expectantly

Prevention of renal stones

Calcium stones may be due to hypercalciuria, which is found in up to 5-


10% of the general population.
• high fluid intake
• low animal protein, low salt diet (a low calcium diet has not been
shown to be superior to a normocalcaemic diet)
• thiazides diuretics (increase distal tubular calcium resorption)

Oxalate stones
• cholestyramine reduces urinary oxalate secretion
• pyridoxine reduces urinary oxalate secretion

Uric acid stones


• allopurinol
• urinary alkalinization e.g. oral bicarbonate

izrotc* •ji
CamScanner
Therapeutic selection

Disease Option

Stone burden of less than 2cm in Lithotripsy


aggregate

Stone burden of less than 2cm in Ureteroscopy


pregnant females

Complex renal calculi and staghorn Percutaneous


calculi nephrolithotomy

Ureteric calculi less than 5mm Manage expectantly

Prevention of renal stones

Calcium stones may be due to hypercalciuria, which is found in up to 5-


10% of the general population.
• high fluid intake
• low animal protein, low salt diet (a low calcium diet has not been
shown to be superior to a normocalcaemic diet)
• thiazides diuretics (increase distal tubular calcium resorption)

Oxalate stones
• cholestyramine reduces urinary oxalate secretion
• pyridoxine reduces urinary oxalate secretion

Uric acid stones


• allopurinol
• urinary alkalinization e.g. oral bicarbonate

izrotc* •ji
CamScanner
G Q15 P3 ©
A 20-year-old Nigerian female presents to the Emergency Department
with chest pain. She reports a long history of fatigue and joint pains.
Examination reveals a pericardial rub and bilateral pitting oedema.
Observations show only that she has a low grade pyrexia 37.7°C and
blood pressure 170 /100 mmHg. Urinalysis shows haematuria and
nephrotic-range proteinuria. A urine pregnancy test is negative. ECG
shows saddle-shaped ST-elevation in all leads. Of interest, her blood
results show:

Urea 8.2 mmol/I

Creatinine 212 pmol/l

eGFR 33ml/m /m 2

Which of the following histopathological findings is most likely on renal


biopsy?

Focal proliferative glomerulonephritis

Membranous glomerulonephritis

Diffuse proliferative glomerulonephritis

Focal segmental glomerulosclerosis

Mesangial proliferative glomerulonephritis

Submit answer

CamScanner
eGFR 33ml/ m/ m2

likely on renal
Which of the following histopathological findings is most
biopsy?

Focal proliferative glomerulonephritis

Membranous glomerulonephritis

Diffuse proliferative glomerulonephritis

Focal segmental glomerulosclerosis

I Mesangial proliferative glomerulonephritis

This question requires you to diagnose the condition and then recall the
associated histopathology. Although they may not be encountered every
day in clinical practice, the glomerulonephritides are favourites of the
MRCP examiners .

The presence of hypertension, kidney injury, proteinuria and haematuria


point towards a nephritic picture of kidney insult .

The multisystem presentation of fever, arthralgia, pericarditis and


nephritis associated with the epidemiological clues ( a young black
female) suggest in this case a diagnosis of systemic lupus
erythematosus (SLE).

The most common histological pattern seen in lupus nephritis is diffuse


proliferative glomerulonephritis.

0 Discuss (7)
* * Improve
i B

Next question >


CamScanner -!
ti f # Discuss (7) Improve

Next question >


Systemic lupus erythematosus: renal
complications

Lupus nephritis is a severe manifestation of systemic lupus


erythematosus (SLE) that can result in end-stage renal disease. SLE
patients should be monitored by performing urinalysis at regular check-
up appointments to rule out proteinuria.

WHO classification
• class I: normal kidney
• class II: mesangial glomerulonephritis
• class III: focal (and segmental) proliferative glomerulonephritis
• class IV: diffuse proliferative glomerulonephritis
• class V: diffuse membranous glomerulonephritis
• class VI: sclerosing glomerulonephritis

Class IV (diffuse proliferative glomerulonephritis) is the most common


and severe form. Renal biopsy characteristically shows the following
findings:
• glomeruli shows endothelial and mesangial proliferation, ' wire-
loop' appearance
• if severe, the capillary wall may be thickened secondary to
immune complex deposition
• electron microscopy shows subendothelial immune complex
deposits
• granular appearance on immunofluorescence

• f 17 * - ram
v ^ • •• %

• fjt #
'\ %
r;
•r
m *
J
s6S&.», f
CamScanner
Features
• nephrotic syndrome
• normotension - hypertension is rare
• highly selective proteinuria
o only intermediate-sized proteins such as albumin and
transferrin leak through the glomerulus
• renal biopsy
o normal glomeruli on light microscopy

o electron microscopy shows fusion of podocytes and

effacement of foot processes

Management
• majority of cases (80%) are steroid-responsive
• cyclophosphamide is the next step for steroid-resistant cases

Prognosis is overall good, although relapse is common. Roughly:


• 1 / 3 have just one episode
• 1 / 3 have infrequent relapses
• 1 / 3 have frequent relapses which stop before adulthood

Next question >

B / a A =' TI" H GO

Save my notes

Search

Search textbook...

CamScanner
G Q17 ©
A 65-year-old gentleman presents to the emergency department with
right upper quadrant pain, fatigue and malaise. He reports that he has
been having the abdominal discomfort for some time but the recent
increase in pain got him concerned. He has a background of recently
diagnosed renal cell carcinoma, hypertension and type-2 diabetes. His
biochemistry results are shown below:

Bilirubin 15 (jmol/ L (3 - 17)

ALP 400 u/L (30 - 100)

ALT 30 u/ L (3 - 40)

YGT 600 u /L (8 - 60)

Albumin 32 g / L (35 - 50)

On examination, he did not appear jaundiced but had right upper


quadrant tenderness on deep palpation with a palpable tender left renal
mass.

Given the clinical and biochemical findings, what is the most likely
cause of his symptoms?

Cholangitis

Paraneoplastic hepatic dysfunction syndrome

Choledocholithiasis

Hepatocellular carcinoma

Hepatorenal failure

Submit answer

CamScanner ^£>.,^11
Paraneoplastic hepatic dysfunction syndrome

Choledocholithiasis

Hepatocellular carcinoma

Hepatorenal failure

Renal cell carcinoma can cause liver dysfunction in particular


cholestasis and hepatosplenomegaly
Important for me Less important

Paraneoplastic syndromes commonly occurring with renal cell


carcinoma can be constitutional symptoms such as fever, cachexia, and
weight loss, ranging to more specific metabolic and biochemical
disturbances like hypercalcemia, non-metastatic liver dysfunction,
amyloidosis etc.

Stauffer 's syndrome is a group of signs / symptoms indicative of liver


dysfunction arising due to underlying renal cell carcinoma.
Biochemically, there is can be an elevated alkaline phosphatase ( ALP),
y -glutamyl transferase (GGT), platelets; prolonged prothrombin time
(PT) with hepatosplenomegaly without hepatic metastasis or jaundice.

Charcot 's triad for cholangitis is the combination of jaundice, fever


(usually rigours) with right upper quadrant abdominal pain. It is caused
by an infection of the bile duct within the liver.

Hepatocellular carcinoma (HCC) is a primary hepatic malignancy from


chronic liver inflammation due to cirrhosis, alcoholism, chronic hepatitis
(B or C). It presents with jaundice, weight loss, malaise, bruising,
abdominal pain and nausea.

Choledocholithiasis is the presence of a stone within the common bile


duct resulting in jaundice with a negative Murphy 's sign.

CamScanner
Choledocholithiasis is the presence of a stone within the common bile
duct resulting in jaundice with a negative Murphy 's sign.

Hepatorenal syndrome (HRS) is a serious condition which arises as a


result of rapid deterioration in renal function in those suffering from
cirrhosis or sudden severe liver failure.

it f # Discuss (2) Improve

Next question >


Renal cell cancer

Renal cell cancer is also known as hypernephroma and accounts for


85% of primary renal neoplasms. It arises from proximal renal tubular
epithelium. The most common histological subtype is clear cell (75 to
85 percent of tumours).

Associations*
• more common in middle-aged men
• smoking
• von Hippel-Lindau syndrome
• tuberous sclerosis

Features
• classical triad: haematuria, loin pain, abdominal mass
• pyrexia of unknown origin
• left varicocele (due to occlusion of left testicular vein)
• endocrine effects: may secrete erythropoietin (polycythaemia),
parathyroid hormone (hypercalcaemia), renin, ACTH
• 25% have metastases at presentation
• paraneoplastic hepatic dysfunction syndrome. Also known as
Stauffer syndrome. Typically presents as
cholestasis/ hepatosplenomegaly. It is thought to be secondary to
increased levels of IL-6

CamScanner
Management
• for confined disease a partial or total nephrectomy depending on
the tumour size
• alpha-interferon and interleukin-2 have been used to reduce
tumour size and also treat patients with metatases
• receptor tyrosine kinase inhibitors (e.g. sorafenib, sunitinib) have
been shown to have superior efficacy compared to interferon-
alpha

\
Coronal CT scan of a middle-aged woman with renal cell cancer.
Note the heterogeneously enhancing mass at the upper pole of
the right kidney

wmi
**frnr CamScanner
Arteriovenous fistulas
Arteriovenous fistulas are direct connections between arteries and veins. They
may occur pathologically but are generally formed surgically to allow access for
haemodialysis.

They are now regarded as the preferred method of access for haemodialysis
due to the lower rates of complications.

The time taken for an arteriovenous fistula to develop is 6 to 8 weeks.

Potential complications include:


• infection
• thrombosis
o may be detected by the absence of a bruit

• stenosis
o may present with acute limb pain
• steal syndrome

From
dialysis
machine Vein Artery
\ t /

\
To
dialysis
nachine

Fistula

CamScanner
Q Q1 P ©
A 4-year -old boy with recurrent urinary tract infections is referred to the
paediatric team due to worries over reflux nephropathy.

Which of the following is the investigation of choice when it comes to


diagnosing this condition?

CT of the kidneys, ureters and bladder

Intravenous pyelogram

Micturating cystography

Renal ultrasound

Renal biopsy

Submit answer

Reference ranges v

Score: 0%

All contents of this site are © 2020 Passmedicine Limited Back to top

CamScanner
A 4-year -old boy with recurrent urinary tract infections is referred to the
paediatric team due to worries over reflux nephropathy.

Which of the following is the investigation of choice when it comes to


diagnosing this condition?

CT of the kidneys, ureters and bladder

Intravenous pyelogram

Micturating cystography

Renal ultrasound

Renal biopsy

Micturating cystography is the investigation of choice for reflux


nephropathy
Importance: 66

This question is asking about the options of imaging in suspected reflux


nephropathy. Reflux nephropathy is commonly caused by recurrent
urinary tract infections, most often in younger children. Urine flows
backwards from the bladder toward the kidneys and over time causes
scarring.

A micturating cystogram is the investigation of choice as this uses


contrast to image this urinary backflow and will show at what level the
urine refluxes to.

While all of the other investigations could be used in imaging of renal


pathology, and some may even be used in the workup of a case like this,
the most definitive and thus best imaging modality for diagnosis, is a
micturating cystogram.

T
CamScanner
*4 t 0 Discuss Improve

Next question >


Reflux nephropathy

Overview
• term used to chronic pyelonephritis secondary to vesico-uretic
reflux
• commonest cause of chronic pyelonephritis
• scarring usually occurs in first 5 years
• strong genetic disposition
• renal scar may produce increased quantities of renin causing
hypertension

Diagnosis
• micturating cystography

Next question >

B / a A a
s ~ TIT SS " H c-o

Save my notes

Search

Search textbook...

Q Google search on "Reflux nephropathy "


CamScanner
© Q4 p ©
A 28-year-old female undergoes a renal transplant for focal segmental
glomerulosclerosis. Within hours of the operation the patient becomes
unwell with features consistent with severe systemic inflammatory
response syndrome. The patient is immediately taken back to theatre
and the transplanted kidney is removed. What type of immunoglobulins
are responsible for the graft rejection?

IgE

IgM

igG

IgD

IgA

Submit answer

Reference ranges v

Score: 100%

>/

2 >/

3 >/

4
CamScanner
A 28-year -old female undergoes a renal transplant for focal segmental
glomerulosclerosis. Within hours of the operation the patient becomes
unwell with features consistent with severe systemic inflammatory
response syndrome. The patient is immediately taken back to theatre
and the transplanted kidney is removed. What type of immunoglobulins
are responsible for the graft rejection?

IgE

IgM

IgG

igD

IgA

Hyperacute graft rejection is due to pre-existent antibodies to HLA


antigens and is therefore IgG mediated

A f # Discuss (1) Improve

Next question >


Renal transplant: HLA typing and graft failure

The human leucocyte antigen (HLA) system is the name given to the
major histocompatibility complex (MHC) in humans. It is coded for on
chromosome 6.

Some basic points on the HLA system


• class 1 antigens include A, B and C. Class 2 antigens include
DRDQ and DR
• when HLA matching for a renal transplant the relative importance
of the HLA antiqens are as follows DR > B > A
CamScanner
Graft survival
• 1 year = 90%, 10 years = 60% for cadaveric transplants
• 1 year = 95%, 10 years = 70% for living-donor transplants

Post-op problems
• ATN of graft
• vascular thrombosis
• urine leakage
• UTI

Hyperacute acute rejection (minutes to hours)


• due to pre-existent antibodies against donor HLA type 1 antigens
(a type II hypersensitivity reaction)
• rarely seen due to HLA matching

Acute graft failure (< 6 months)


• usually due to mismatched HLA. Cell-mediated (cytotoxic T cells)
• other causes include cytomegalovirus infection
• may be reversible with steroids and immunosuppressants

Causes of chronic graft failure (> 6 months)


• both antibody and cell mediated mechanisms cause fibrosis to
the transplanted kidney (chronic allograft nephropathy)
• recurrence of original renal disease (MCGN > IgA > FSGS)

Next question >


B / a A =' TI'*’ Mw E co

Save my notes

CamScanner
' * ' El
o° •:< ? ® O i
^ iaiii -
OUJIS o

I PassMedicine

© Q7 p o
A 43-year-old man has a work-up for hypertension. He has found to
have blood + on a urine dipstick of a freshly voided sample. Which one
of the following may account for this finding?

Smoking

Exercise

Obesity

Eating red meat the previous day

Use of ramipril

Submit answer

Reference ranges v

Score: 66.7%

1 V
2 V
3 >/

4 X
5 >/
6 X
7
<] o
CamScanner
© Q7 p ©
A 43-year-old man has a work-up for hypertension. He has found to
have blood + on a urine dipstick of a freshly voided sample. Which one
of the following may account for this finding?

Smoking

Exercise

Obesity

Eating red meat the previous day

Use of ramipril

i4 f # Discuss (2) Improve

Next question >


Haematuria

The management of patients with haematuria is often difficult due to


the absence of widely followed guidelines. It is sometimes unclear
whether patients are best managed in primary care, by urologists or by
nephrologists.

The terminology surrounding haematuria is changing. Microscopic or


dipstick positive haematuria is increasingly termed non-visible
hppmptnrip VA/ hilQt mprrncr'nnir hppmptnrip iQ fprmprl viQihlp
CamScanner
Next question >
Haematuria

The management of patients with haematuria is often difficult due to


the absence of widely followed guidelines. It is sometimes unclear
whether patients are best managed in primary care, by urologists or by
nephrologists.

The terminology surrounding haematuria is changing. Microscopic or


dipstick positive haematuria is increasingly termed non-visible
haematuria whilst macroscopic haematuria is termed visible
haematuria. Non-visible haematuria is found in around 2.5% of the
population.

Causes of transient or spurious non-visible haematuria


• urinary tract infection
• menstruation
• vigorous exercise (this normally settles after around 3 days)
• sexual intercourse

Causes of persistent non-visible haematuria


• cancer (bladder, renal, prostate)
• stones
• benign prostatic hyperplasia
• prostatitis
• urethritis e.g. Chlamydia
• renal causes: IgA nephropathy, thin basement membrane disease

Spurious causes - red/ orange urine, where blood is not present on


dipstick
• foods: beetroot, rhubarb
• drugs: rifampicin, doxorubicin

CamScanner
Management

Current evidence does not support screening for haematuria. The


incidence of non-visible haematuria is similar in patients taking
aspirin/ warfarin to the general population hence these patients should
also be investigated.

Testing
• urine dipstick is the test of choice for detecting haematuria
• persistent non-visible haematuria is often defined as blood being
present in 2 out of 3 samples tested 2-3 weeks apart
• renal function, albumin:creatinine (ACR) or protein:creatinine ratio
(PCR) and blood pressure should also be checked
• urine microscopy may be used but time to analysis significantly
affects the number of red blood cells detected

NICE urgent cancer referral guidelines were updated in 2015.

Urgent referral (i.e. within 2 weeks)

Aged >= 45 years AND:


• unexplained visible haematuria without urinary tract infection, or
• visible haematuria that persists or recurs after successful
treatment of urinary tract infection

Aged >= 60 years AND have unexplained nonvisible haematuria and


either dysuria or a raised white cell count on a blood test

Non- urgent referral

Aged 60 >= 60 years with recurrent or persistent unexplained urinary


tract infection

Since the investigation (or not) of non-visible haematuria is such as a


common dilemma a number of guidelines have been published. They
generally agree with NICE guidance, of note:
A r\ o +i i i -J
/
- -
+U / » / » AO t t i l+U r> /> tr I I f i ir* S tr\

CamScanner
Testing
• urine dipstick is the test of choice for detecting haematuria
• persistent non-visible haematuria is often defined as blood being
present in 2 out of 3 samples tested 2-3 weeks apart
• renal function, albumin:creatinine (ACR) or protein:creatinine ratio
(PCR) and blood pressure should also be checked
• urine microscopy may be used but time to analysis significantly
affects the number of red blood cells detected

NICE urgent cancer referral guidelines were updated in 2015.

Urgent referral (i.e. within 2 weeks)

Aged >= 45 years AND:


• unexplained visible haematuria without urinary tract infection, or
• visible haematuria that persists or recurs after successful
treatment of urinary tract infection

Aged >= 60 years AND have unexplained nonvisible haematuria and


either dysuria or a raised white cell count on a blood test

Non- urgent referral

Aged 60 >= 60 years with recurrent or persistent unexplained urinary


tract infection

Since the investigation (or not) of non-visible haematuria is such as a


common dilemma a number of guidelines have been published. They
generally agree with NICE guidance, of note:
• patients under the age of 40 years with normal renal function, no
proteinuria and who are normotensive do not need to be referred
and may be managed in primary care

Next question >


CamScanner
G Q8 PJ ©
Which one of the following is least associated with retroperitoneal
fibrosis?

Riedel 's thyroiditis

Previous radiotherapy

Inflammatory abdominal aortic aneurysm

Methysergide

Sulphonamides

Submit answer

Reference ranges v

Score: 71.4%

1 V
2 >/

3
4 X

CamScanner
Which one of the following is least associated with retroperitoneal
fibrosis?

Riedel's thyroiditis

Previous radiotherapy

Inflammatory abdominal aortic aneurysm

Methysergide

Sulphonamides

4 8
f # Discuss Improve

Next question >


Retroperitoneal fibrosis

Lower back / flank pain is the most common presenting feature. Fever
and lower limb oedema is also seen in some patients.

Associations
• Riedel's thyroiditis
• previous radiotherapy
• sarcoidosis
• inflammatory abdominal aortic aneurysm
• drugs: methysergide

Next question >


D I A M
1
— HP T
CamScanner
© Q9 \C3 ©
A 61-year -old woman is evaluated for painful skin lesions that have
developed over the past 1-2 weeks. She has longstanding end- stage
renal disease, due to diabetic nephropathy, for which she undergoes
hemodialysis 3 times a week. Temperature is 37 C (98.6 F), blood
pressure is 159 / 79 mmHg, pulse is 97/ min, and respirations are
16/ min. Laboratory results show elevated blood urea nitrogen, elevated
serum creatinine, and elevated serum calcium. Punch biopsy of a lesion
shows calcification of the middle layer of the arterioles, subintimal
fibrosis, and thrombotic occlusion without vasculitis.

Which of the following is most likely diagnosis?

Calciphylaxis

Progressive systemic sclerosis

Nephrogenic systemic fibrosis

Kyrle disease

Glucagonoma

Submit answer

Reference ranges v

CamScanner
Q9 p

A 61-year-old woman is evaluated for painful skin lesions that have


developed over the past 1-2 weeks. She has longstanding end-stage
renal disease, due to diabetic nephropathy, for which she undergoes
hemodialysis 3 times a week. Temperature is 37 C (98.6 F), blood
pressure is 159 /79 mmHg, pulse is 97/ min, and respirations are
16/ min. Laboratory results show elevated blood urea nitrogen, elevated
serum creatinine, and elevated serum calcium. Punch biopsy of a lesion
shows calcification of the middle layer of the arterioles, subintimal
fibrosis, and thrombotic occlusion without vasculitis.

Which of the following is most likely diagnosis?

Calciphylaxis

Progressive systemic sclerosis

Nephrogenic systemic fibrosis

Kyrle disease

Glucagonoma

Calciphylaxis lesion are intensely painful, purpuric patches with


an area of black necrotic tissue that may form bullae, ulcerate,
and leave a hard, firm eschar
Important for me Less important

This patient has calciphylaxis (also known as calcific uremic


arteriolopathy), which is characterized by systemic arteriolar
calcification and soft-tissue calcium deposition with local ischemia and
necrosis. Calciphylaxis presents with painful subcutaneous plaques and
nodules that can be violaceous or purpuric. The nodules can progress
to ischemic or necrotic ulcers, become infected, and lead to sepsis.

CamScanner
Calciphylaxis lesion are intensely painful, purpuric patches with
an area of black necrotic tissue that may form bullae, ulcerate,
and leave a hard, firm eschar
Importance: 94

This patient has calciphylaxis (also known as calcific uremic


arteriolopathy), which is characterized by systemic arteriolar
calcification and soft-tissue calcium deposition with local ischemia and
necrosis. Calciphylaxis presents with painful subcutaneous plaques and
nodules that can be violaceous or purpuric. The nodules can progress
to ischemic or necrotic ulcers, become infected, and lead to sepsis.

Progressive systemic sclerosis is an autoimmune disease with


cutaneous and systemic findings. The cutaneous findings are
thickening of the skin which results in lack of joint mobility, dilated
periungual capillary loops.

Nephrogenic systemic fibrosis is characterized by yellowish, thickened


papules and nodules with progressive skin tightening and sclerosis. It is
seen in patients undergoing dialysis for end-stage kidney disease who
were exposed to gadolinium containing contrast dye.

Kyrle disease is most frequently seen in persons with diabetes and end-
stage kidney disease. It can present with an individual papule or
numerous widespread hyperpigmented papules. The papules have an
umbilicated central core.

Glucagonoma is characterized by glucagon overproduction, weight loss,


diabetes mellitus, normochromic and normocytic anemia, and
necrolytic migratory erythema.

BA f # Discuss (1) Improve

CamScanner
Q10 p

Each one of the following is a cause of nephrogenic diabetes insipidus,


except:

Hypocalcaemia

Sickle-cell anaemia

Lithium

Hypokalaemia

Demeclocycline

Submit answer

Reference ranges v

Score: 66.7%

1 >/

2 V
3 >/

4 X
5 V
6 X
7
Q
CamScanner
Q Q10 x ©
Each one of the following is a cause of nephrogenic diabetes insipidus,
except:

Hypocalcaemia

Sickle-cell anaemia

Lithium

Hypokalaemia

Demeclocycline

B6 t # Discuss Improve

Next question >


Diabetes insipidus

Diabetes insipidus (Dl) is a condition characterised by either a


deficiency of antidiuretic hormone, ADH, (cranial Dl) or an insensitivity
to antidiuretic hormone (nephrogenic Dl).

HaiiQPQ nf rranipl HI

CamScanner
Diabetes insipidus

Diabetes insipidus (Dl) is a condition characterised by either a


deficiency of antidiuretic hormone, ADH, (cranial Dl) or an insensitivity
to antidiuretic hormone (nephrogenic Dl).

Causes of cranial Dl
• idiopathic
• post head injury
• pituitary surgery
• craniopharyngiomas
• histiocytosis X
• DIDMOAD is the association of cranial Diabetes Insipidus,
Diabetes Mellitus, Optic Atrophy and Deafness (also known as
Wolfram's syndrome)
• haemochromatosis

Causes of nephrogenic Dl
• genetic: the more common form affects the vasopression (ADH)
receptor, the less common form results from a mutation in the
gene that encodes the aquaporin 2 channel
• electrolytes: hypercalcaemia, hypokalaemia
• drugs: demeclocycline, lithium
• tubulo-interstitial disease: obstruction, sickle-cell, pyelonephritis

Features
• polyuria
• polydipsia

Investigation
• high plasma osmolality, low urine osmolality
• a urine osmolality of >700 mOsm/ kg excludes diabetes insipidus
• water deprivation test

CamScanner
• DIDMOAD is the association ot cranial Diabetes Insipidus,
Diabetes Mellitus, Optic Atrophy and Deafness (also known as
Wolfram's syndrome)
• haemochromatosis

Causes of nephrogenic Dl
• genetic: the more common form affects the vasopression (ADH)
receptor, the less common form results from a mutation in the
gene that encodes the aquaporin 2 channel
• electrolytes: hypercalcaemia, hypokalaemia
• drugs: demeclocycline, lithium
• tubulo-interstitial disease: obstruction, sickle-cell, pyelonephritis

Features
• polyuria
• polydipsia

Investigation
• high plasma osmolality, low urine osmolality
• a urine osmolality of >700 mOsm/kg excludes diabetes insipidus
• water deprivation test

Management
• nephrogenic diabetes insipidus: thiazides, low salt / protein diet
- central diabetes insipidus can be treated with desmopressin

Next question >

B / B A HI
TI~ 0 co

Save my notes

CamScanner
© Q11 p ©
A 55-year-old woman is seen in the Emergency Department, she was
previously well except poorly controlled hypertension and a 20 pack-
year smoking history. She has developed sudden shortness of breath
with a productive clear cough. On examination you find bilateral
dullness to percussion with fine crackles in both bases, no other
findings.

Chest radiograph shows pulmonary oedema but the echocardiogram


shows no left ventricular impairment. The consultant asks you to order
an ultrasound of her kidney ureter and bladder.

What is he suspecting this investigation will show?

Malignancy

Asymmetric kidneys

Hydronephrosis

Nephrolithiasis

Normal kidneys

Submit answer

Reference ranges v

CamScanner
© Q11 p ©
A 55-year-old woman is seen in the Emergency Department, she was
previously well except poorly controlled hypertension and a 20 pack-
year smoking history. She has developed sudden shortness of breath
with a productive clear cough. On examination you find bilateral
dullness to percussion with fine crackles in both bases, no other
findings.

Chest radiograph shows pulmonary oedema but the echocardiogram


shows no left ventricular impairment . The consultant asks you to order
an ultrasound of her kidney ureter and bladder.

What is he suspecting this investigation will show ?

Malignancy

Asymmetric kidneys

Hydronephrosis

Nephrolithiasis

Normal kidneys

Young female, hypertension and asymmetric kidneys


fibromuscular dysplasia
Important for me Less important

This woman has presented to the emergency department with rapid


onset (flash) pulmonary oedema.

She is female vouna. and a smoker all risk factors for fibromuscular
, ,

CamScanner
This woman has presented to the emergency department with rapid
onset (flash) pulmonary oedema.

She is female, young, and a smoker, all risk factors for fibromuscular
dysplasia. Her hypertension is poorly controlled which suggests there
may be an underlying cause for the hypertension.

Recurrent, flash pulmonary oedema in renal vascular disease is


probably a consequence of fluid retention, and diastolic ventricular
dysfunction.

Renal vascular disease will present on ultrasound as asymmetric


kidneys with the affected side being smaller. Doppler ultrasound would
also show disturbance in renal blood flow.

Renal malignancy would more likely present gradually and with


haematuria, an abdominal mass, lethargy, anorexia and weight loss.

Asymmetric kidneys are the finding in fibromuscular dysplasia.

The presentation of this woman is suggestive of renal vascular disease,


this would not cause hydronephrosis. If she had hydronephrosis she
would be more likely to present with symptoms of retention or
symptoms of kidney stones.

Kidney stones would be more likely to present with symptoms such as


back, abdominal or groin pain, haematuria, nausea, pyrexia or sweating.

If the consultant suspected the investigation would be normal he would


hopefully not order it.

CamScanner
U I 5L U 6 A I ) MI i p i u v e
^
Next question >
Fibromuscular dysplasia

Renal artery stenosis secondary to atherosclerosis accounts for around


90% of renal vascular disease, with fibromuscular dysplasia being the
most common cause of the remaining 10%.

Epidemiology
• 90% of patients are female

Features
• hypertension
• chronic kidney disease or more acute renal failure e.g. secondary
to ACE-inhibitor initiation
• 'flash' pulmonary oedema

Next question >

B / a A • =Z ^ TI" H c-D

Save my notes

Search

Search textbook...

Q Google search on "Fibromuscular dysplasia"


CamScanner
G Q12 P ©
A 32-year-old woman comes to the immunology clinic for review. She
has a history of SLE and stopped her medication prior to getting
pregnant. She is now approaching 16 weeks gestation. Unfortunately
her joint pains have returned, creatinine has risen from 90 pmol/ l to 146
pmol/ l, with blood and proteinuria, despite high dose prednisolone.

Which of the following is the most appropriate next step?

Azathioprine

Ciclosporin

Cyclophosphamide

Methotrexate

Mycophenolate

Submit answer

Reference ranges v

Score: 63.6%

CamScanner
UT7 x7
A 32-year -old woman comes to the immunology clinic for review. She
has a history of SLE and stopped her medication prior to getting
pregnant. She is now approaching 16 weeks gestation. Unfortunately
her joint pains have returned, creatinine has risen from 90 pmol/ l to 146
pmol/ l, with blood and proteinuria, despite high dose prednisolone.

Which of the following is the most appropriate next step?

Azathioprine

Ciclosporin

Cyclophosphamide

Methotrexate

Mycophenolate

A large body of evidence from the use of azathioprine in pregnancy for


the treatment of both rheumatological conditions and inflammatory
bowel disease, supports its use. Although it is less effective in the
management of SLE with renal disease versus other options, balance of
benefit risk makes it the preferred intervention.

Ciclosporin appears to be associated with premature delivery and low


birth weight, although it does not seem to be associated with
malformations, this drives its use as an alternative to azathioprine in
patients who fail to gain control of their disease. Cyclophosphamide,
methotrexate and mycophenolate are all contraindicated for use in
pregnancy.

http:// www.ncbi.nlm.nih.gov / pmc / articles / PMC3237512/

lA Discuss ( A ) Imnrove
CamScanner
Systemic lupus erythematosus: renal
complications

Lupus nephritis is a severe manifestation of systemic lupus


erythematosus (SLE) that can result in end-stage renal disease. SLE
patients should be monitored by performing urinalysis at regular check-
up appointments to rule out proteinuria.

WHO classification
• class I: normal kidney
• class II: mesangial glomerulonephritis
• class III: focal (and segmental) proliferative glomerulonephritis
• class IV: diffuse proliferative glomerulonephritis
• class V: diffuse membranous glomerulonephritis
• class VI: sclerosing glomerulonephritis

Class IV (diffuse proliferative glomerulonephritis) is the most common


and severe form. Renal biopsy characteristically shows the following
findings:
• glomeruli shows endothelial and mesangial proliferation, ' wire-
loop' appearance
• if severe, the capillary wall may be thickened secondary to
immune complex deposition
• electron microscopy shows subendothelial immune complex
deposits
• granular appearance on immunofluorescence

CamScanner
© Q13 p ©
A 33-year -old is admitted to the Emergency Department with suspected
renal colic. He has a ultrasound that shows a probable stone in the left
ureter. What is the most appropriate next step with respect to imaging?

Non-contrast CT (NCCT)

Micturating cystourethrogram

Intravenous urography (IVU)

Plain radiography KUB

MRI

Submit answer

Reference ranges v

Score: 58.3%

1 V
2 V
3 V

CamScanner
© Q13 p ©
A 33-year -old is admitted to the Emergency Department with suspected
renal colic. He has a ultrasound that shows a probable stone in the left
ureter. What is the most appropriate next step with respect to imaging?

Non-contrast CT (NCCT)

Micturating cystourethrogram

Intravenous urography (IVU)

Plain radiography KUB

MRI

The 2015 BAUS guidelines state:

Following initial US assessment, NCCT should be used to confirm


stone diagnosis in patients with acute flank pain, because it is
superior to IVU.

B6 f # Discuss (2) Improve

CamScanner
A 54-year-old man who has end stage diabetic nephropathy is being
assessed for a renal transplant. When assessing the HLA matching
between donor and recipient what is the most important HLA antigen to
match?

DP

O DR

Submit answer

Reference ranges

Score: 61.5%

1 V
2 V
3
4 X
5 >/

6 X
CamScanner
A 54-year-old man who has end stage diabetic nephropathy is being
assessed for a renal transplant . When assessing the HLA matching
between donor and recipient what is the most important HLA antigen to
match?

DP

DR

Renal transplant HLA matching - DR is the most important


Importance: 36

4 Discuss (2) Improve


i
* ^
Next question >
Renal transplant: HLA typing and graft failure

The human leucocyte antigen (HLA) system is the name given to the
major histocompatibility complex (MHC) in humans. It is coded for on
chromosome 6.

Some basic points on the HLA system


• class 1 antigens include A, B and C. Class 2 antigens include
DRDQ and DR
• when HLA matching for a renal transplant the relative importance
CamScanner
© Q16 p

A 39-year -old woman with systemic lupus erythematosus presents for


review in the rheumatology clinic. On examination mild pedal oedema is
noted and the blood pressure is slightly raised at 160 / 92 mmHg.
Dipstick urine on arrival at clinic shows protein ++, blood+++. Further
investigations reveal the following

Bicarbonate 22 mmol/ l

Urea 7.1 mmol /l

Creatinine 134 pmol/l

24-hour urinary protein 2.6 g

What is the renal biopsy most likely to show ?

Diffuse proliferative glomerulonephritis

Mesangiocapillary glomerulonephritis

Rapidly progressive glomerulonephritis

Membranous glomerulonephritis

Minimal change disease

Submit answer

CamScanner
Bicarbonate 22 mmol/ I

Urea 7.1 mmol / l

Creatinine 134 (jmol/ l

24-hour urinary protein 2.6 g

What is the renal biopsy most likely to show ?

Diffuse proliferative glomerulonephritis

Mesangiocapillary glomerulonephritis

Rapidly progressive glomerulonephritis

Membranous glomerulonephritis

Minimal change disease

Diffuse proliferative glomerulonephritis is the most common and


severe form of renal disease in SLE patients
Importance: 87

Diffuse proliferative glomerulonephritis is the most common and severe


form of renal disease in SLE patients. The following features are
supportive of this diagnosis:
• haematuria
• proteinuria
• oedema
• hypertension

«4 ? # Discuss (3) Improve

CamScanner ^£>.,^11
Which one of the following statements is true regarding autosomal
recessive polycystic kidney disease?

Onset is typically in the third decade

Liver involvement is rare

Is due to a defect on chromosome 16

More common than autosomal dominant polycystic kidney disease

May be diagnosed on prenatal ultrasound

A "t # Discuss (1) Improve

Next question >


ARPKD

Autosomal recessive polycystic kidney disease (ARPKD) is much less


common than autosomal dominant disease (ADPKD). It is due to a
defect in a gene located on chromosome 6 which encodes fibrocystin, a
protein important for normal renal tubule development.

Diagnosis may be made on prenatal ultrasound or in early infancy with


abdominal masses and renal failure. Newborns may also have features
consistent with Potter's syndrome secondary to oligohydramnios. End-
stage renal failure develops in childhood. Patients also typically have
liver involvement, for example portal and interlobular fibrosis.

Renal biopsy typically shows multiple cylindrical lesions at right angles


to the cortical surface.
CamScanner
I
* f # Discuss (1) Improve

Next question >

ARPKD

Autosomal recessive polycystic kidney disease (ARPKD) is much less


common than autosomal dominant disease (ADPKD). It is due to a
defect in a gene located on chromosome 6 which encodes fibrocystin, a
protein important for normal renal tubule development.

Diagnosis may be made on prenatal ultrasound or in early infancy with


abdominal masses and renal failure. Newborns may also have features
consistent with Potter 's syndrome secondary to oligohydramnios. End-
stage renal failure develops in childhood. Patients also typically have
liver involvement, for example portal and interlobular fibrosis.

Renal biopsy typically shows multiple cylindrical lesions at right angles


to the cortical surface.

Next question >

B / a A = - xr H - £D c-o

Save my notes

CamScanner -;
Q2 p

A mutation in the gene that encodes aquaporin 2 is most likely to result


in:

Histiocytosis

Alport 's syndrome

Syndrome of inappropriate ADH secretion

Diabetes insipidus

Medullary sponge kidney

Submit answer

Reference ranges v

Score: 100%

1 >/

All contents of this site are © 2020 Passmedicine Limited Back to top

CamScanner
Q2 X ©
A mutation in the gene that encodes aquaporin 2 is most likely to result
in:

Histiocytosis

Alport ’s syndrome

Syndrome of inappropriate ADH secretion

Diabetes insipidus

Medullary sponge kidney

Nephrogenic diabetes insipidus may be caused genetic


mutations:
• the more common form affects the vasopression (ADH)
receptor
• the less common form results from a mutation in the gene
that encodes the aquaporin 2 channel

Importance: 82

4
B f
8
# Discuss (1) Improve

Next question >


Diabetes insipidus
CamScanner
G Q3 P O
82-year-old female with a background of rheumatoid arthritis on
maintenance dose prednisolone and sulfasalazine is admitted with
deteriorating renal function. Urine dip reveals protein +++. She has
previously suffered from a distal radial fracture and is currently
complaining of numbness and tingling down the lateral 3 1 / 2 digits of
the opposite hand.

Renal ultrasound reveals bilaterally enlarged kidneys and a


subcutaneous abdominal fat biopsy is positive after Congo red staining.
What is the most likely unifying diagnosis?

Diabetes

AL amyloidosis

AA amyloidosis

Waldenstrom macroglobulinaemia

Systemic lupus erythematosus

Submit answer

Reference ranges

Score: 50%

CamScanner
© Q3 X P ©
82-year-old female with a background of rheumatoid arthritis on
maintenance dose prednisolone and sulfasalazine is admitted with
deteriorating renal function. Urine dip reveals protein +++. She has
previously suffered from a distal radial fracture and is currently
complaining of numbness and tingling down the lateral 3 1/ 2 digits of
the opposite hand.

Renal ultrasound reveals bilaterally enlarged kidneys and a


subcutaneous abdominal fat biopsy is positive after Congo red staining.
What is the most likely unifying diagnosis?

Diabetes

AL amyloidosis

AA amyloidosis

Waldenstrom macroglobulinaemia

Systemic lupus erythematosus

Secondary AA amyloidosis is most likely given the background of


rheumatoid arthritis (the second most common cause after juvenile
idiopathic arthritis in the UK), heavy proteinuria, and positive Congo red
staining.

Diabetes would be a possibility, however, it is not associated with a


positive Congo red staining. AL amyloidosis tends to be associated with
an underlying haematological condition as opposed to an inflammatory
problem. Again, Waldenstrom macroglobulinaemia and systemic lupus
erythematosus would not cause a positive Congo red staining.

>4 f # Discuss (3) Improve


CamScanner
G Q1 P ©
A 55-year -old presents to the Emergency Department with shortness-of -
breath since the morning. Last month he was admitted following an
inferior myocardial infarction. He was started on aspirin, atorvastatin,
lisinopril and bisoprolol. An echocardiogram performed following the
myocardial infarction showed normal left ventricular function. He is still
a smoker despite repeated attempts to give up.

Examination today reveals bibasal crackles whilst the chest x-ray shows
upper lobe diversion and perihilar shadowing. The ECG and cardiac
enzymes are normal.

What is the most likely cause of his breathlessness?

Infective endocarditis

Phaeochromocytoma

Fibromuscular dysplasia

Renal artery stenosis

Anterior myocardial infarction

Submit answer

CamScanner
Flash pulmonary oedema, U&Es worse on ACE inhibitor,
asymmetrical kidneys
angiography
- renal artery stenosis - do MR

Important for me Less important

Renal artery stenosis may cause sudden onset or 'flash' pulmonary


oedema. A myocardial infarction is unlikely given the normal ECG and
cardiac enzymes. Chest pain would also be expected in a 55-year -old
patient with no history of diabetes. Fibromuscular dysplasia is generally
seen in young woman.

-? # Discuss (6) Improve

Next question >


Renal vascular disease

Renal vascular disease is most commonly due to atherosclerosis (>


95% of patients). It is associated with risk factors such as smoking and
hypertension that cause atheroma elsewhere in the body. It may present
as hypertension, chronic renal failure or ' flash' pulmonary oedema. In
younger patients however fibromuscular dysplasia (FMD) needs to be
considered. FMD is more common in young women and
characteristically has a 'string of beads' appearance on angiography.
Patients respond well to balloon angioplasty

Investigation
• MR angiography is now the investigation of choice
• CT angiography
• conventional renal angiography is less commonly performed used
nowadays, but may still have a role when planning surgery

Next question >


CamScanner
© Q4 p

A 62-year-old man with a diabetic nephropathy and hypertension is


reviewed. His current medication is insulin, bendroflumethiazide,
ramipril and amlodipine. On examination blood pressure is 144/ 78
mmHg. Blood tests reveal the following:

Na+ 139 mmol/ l

K+ 4.9 mmol/l

Urea 12.8 mmol/l

Creatinine 215 pmol /l

eGFR 29 ml/ min

Renal function was similar to 3 months ago. What is the most


appropriate action?

No change to his medication

Switch bendroflumethiazide to furosemide

Add a beta-blocker

Add spironolactone

Stop ramipril

Submit answer

Reference ranges v

CamScanner
© Q4 X p

A 62-year-old man with a diabetic nephropathy and hypertension is


reviewed. His current medication is insulin, bendroflumethiazide,
ramipril and amlodipine. On examination blood pressure is 144/ 78
mmHg. Blood tests reveal the following:

Na + 139 mmol / l

K+ 4.9 mmol/l

Urea 12.8 mmol/ l

Creatinine 215 (jmol / l

eGFR 29 ml / min

Renal function was similar to 3 months ago. What is the most


appropriate action?

No change to his medication

Switch bendroflumethiazide to furosemide

Add a beta-blocker

Add spironolactone

Stop ramipril

As the eGFR is 29 ml/ min switching bendroflumethiazide to furosemide


would be the next step in controlling his blood pressure. Please see the
guidelines in the external links section

si t # Discuss (5) Improve


CamScanner
R6 # Discuss (5) Improve

Next question >


Chronic kidney disease: hypertension

The majority of patients with chronic kidney disease (CKD) will require
more than two drugs to treat hypertension. ACE inhibitors are first line
and are particularly helpful in proteinuric renal disease (e.g. diabetic
nephropathy). As these drugs tend to reduce filtration pressure a small
fall in glomerular filtration pressure (GFR) and rise in creatinine can be
expected. NICE suggest that a decrease in eGFR of up to 25% or a rise
in creatinine of up to 30% is acceptable, although any rise should
prompt careful monitoring and exclusion of other causes (e. g. NSAIDs) .
A rise greater than this may indicate underlying renovascular disease.

Furosemide is useful as a anti-hypertensive in patients with CKD,


particularly when the GFR falls to below 45 ml / min*. It has the added
benefit of lowering serum potassium. High doses are usually required. If
the patient becomes at risk of dehydration (e.g. Gastroenteritis) then
consideration should be given to temporarily stopping the drug

*the NKF K / DOQI guidelines suggest a lower cut-off of less than 30


ml/ min

Next question >


B I B A El ^ TP HP E -
cD

Save my notes

CamScanner
© Q5 p

A 45-year-old presents to the Emergency Department with chest pain.


An ECG shows anterior ST elevation and he is thrombolysed with
alteplase. His chest pain settles and he is started on aspirin,
atorvastatin, bisoprolol and ramipril. Four days later his blood results
are as follows:

Urea 22 mmol/l

Creatinine 277 pmol/l

What is the most likely cause for the deterioration in renal function?

Renal artery stenosis

NSAID related nephropathy

Statin nephropathy

Dressler 's syndrome

Haemorrhage into renal cyst

Submit answer

Reference ranges v

Score: 50%

CamScanner
Q Q5 >/ ©
A 45-year-old presents to the Emergency Department with chest pain.
An ECG shows anterior ST elevation and he is thrombolysed with
alteplase. His chest pain settles and he is started on aspirin,
atorvastatin, bisoprolol and ramipril. Four days later his blood results
are as follows:

Urea 22 mmol/I

Creatinine 277 pmol/ l

What is the most likely cause for the deterioration in renal function?

Renal artery stenosis

NSAID related nephropathy

Statin nephropathy

Dressler 's syndrome

Haemorrhage into renal cyst

Flash pulmonary oedema, U&Es worse on ACE inhibitor,


asymmetrical kidneys
angiography
- renal artery stenosis - do MR

Important for me Less important

There is likely underlying renal artery stenosis revealed by the addition


of an ACE inhibitor. Risk factors such as hypertension and
hyperlipidaemia which have contributed to the development of his
ischaemic heart disease also put him at risk of renal vascular disease

A f # Discuss (6) Improve

CamScanner
Q Q2 P3 ©
A 19-year -old woman is referred to the nephrology department after
experiencing several episodes of visible haematuria. There is no history
of abdominal or loin pain. These typically seem to occur within a day or
two of developing tonsillitis. Blood pressure is 148 / 90 mmHg.

Urine dipstick is normal. Blood tests show the following:

Na + 142 mmol/l

K+ 4.1 mmol/ l

Bicarbonate 24 mmol/l

Urea 3.5 mmol/l

Creatinine 71 pmol/l

Given the likely diagnosis, which one of the following is a marker of


poor prognosis?

Female gender

Hypertension

Frank haematuria

Absence of proteinuria

Development of the disease before the age of 20 years

Submit answer

CamScanner
Q2 X |o V
A 19-year -old woman is referred to the nephrology department after
experiencing several episodes of visible haematuria. There is no history
of abdominal or loin pain. These typically seem to occur within a day or
two of developing tonsillitis. Blood pressure is 148 / 90 mmHg.

Urine dipstick is normal. Blood tests show the following:

Na + 142 mmol / l

K+ 4.1 mmol /l

Bicarbonate 24 mmol/ l

Urea 3.5 mmol /l

Creatinine 71 Mmol /l

Given the likely diagnosis, which one of the following is a marker of


poor prognosis?

Female gender

Hypertension

Frank haematuria

Absence of proteinuria

Development of the disease before the age of 20 years

t # Discuss (1) Improve

CamScanner
Q3 P3

Autosomal dominant polycystic kidney disease type 2 is associated


with a gene defect in:

Chromosome 4

Chromosome 8

Chromosome 12

Chromosome 16

Chromosome 20

Submit answer

Reference ranges v

Score: 0%

1 X
2 X
3

i : Dorl/ tr» r
-
CamScanner -; Lig b
G Q3 V P ©
Autosomal dominant polycystic kidney disease type 2 is associated
with a gene defect in:

Chromosome 4

Chromosome 8

Chromosome 12

Chromosome 16

Chromosome 20

ADPKD type 2 = chromosome 4 = 15% of cases


Importance: 28

it Bf # Discuss (1) Improve

Next question >


ADPKD
CamScanner -; a^ c,. ,,AAll
.
Autosomal dominant polycystic kidney disease (ADPKD) is the most
common inherited cause of kidney disease, affecting 1 in 1,000
Caucasians. Two disease loci have been identified, PKD1 and PKD 2,
which code for polycystin-1 and polycystin-2 respectively

ADPKD type 1 ADPKD type 2

85% of cases 15% of cases

Chromosome 16 Chromosome 4

Presents with renal failure earlier

The screening investigation for relatives is abdominal ultrasound:

Ultrasound diagnostic criteria (in patients with positive family history)


• two cysts, unilateral or bilateral, if aged < 30 years
• two cysts in both kidneys if aged 30-59 years
• four cysts in both kidneys if aged > 60 years

Management

For select patients, tolvaptan (vasopressin receptor 2 antagonist) may


be an option. NICE recommended it as an option for treating ADPKD in
adults to slow the progression of cyst development and renal
insufficiency only if:
• they have chronic kidney disease stage 2 or 3 at the start of
treatment
• there is evidence of rapidly progressing disease and
• the company provides it with the discount agreed in the patient
access scheme.

CamScanner
Q Q4 ©
A patient with type 1 diabetes mellitus is reviewed in the nephrology
outpatient clinic. He is known to have stage 1 diabetic nephropathy.
Which of the following best describes his degree of renal involvement ?

Latent phase

Hyperfiltration

End-stage renal failure

Overt nephropathy

Microalbuminuria

Submit answer

Reference ranges v

Score: 33.3%

1 X
2 X
3 v>
4

CamScanner
Hyperfiltration

End-stage renal failure

Overt nephropathy

Microalbuminuria

For the purposes of the MRCR increase in the glomerular filtration rate
(GFR) is most characteristic of stage 1 diabetic nephropathy. It is
however known that elevation of the GFR usually persists into stage 2

4 t # Discuss (1) Improve

Next question >


Diabetic nephropathy: stages

Diabetic nephropathy may be classified as occurring in five stages*:

Stage 1
• hyperfiltration: increase in GFR
• may be reversible

Stage 2 (silent or latent phase)


• most patients do not develop microalbuminuria for 10 years
• GFR remains elevated

Stage 3 (incipient nephropathy)


• microalbuminuria (albumin excretion of 30 - 300 mg/ day, dipstick
negative)
CamScanner
Diabetic nephropathy may be ciassitied as occurring in rive stages*:

Stage 1
• hyperfiltration: increase in GFR
• may be reversible

Stage 2 (silent or latent phase)


• most patients do not develop microalbuminuria for 10 years
• GFR remains elevated

Stage 3 (incipient nephropathy)


• microalbuminuria (albumin excretion of 30 - 300 mg/ day, dipstick
negative)

Stage 4 (overt nephropathy)


• persistent proteinuria (albumin excretion > 300 mg/ day, dipstick
positive)
• hypertension is present in most patients
• histology shows diffuse glomerulosclerosis and focal
glomerulosclerosis (Kimmelstiel-Wilson nodules)

Stage 5
• end-stage renal disease, GFR typically < 10ml / min
• renal replacement therapy needed

The timeline given here is for type 1 diabetics. Patients with type 2
diabetes mellitus (T 2DM) progress through similar stages but in a
different timescale - some T 2DM patients may progress quickly to the
later stages

Next question >


^-
CamScanner -; l s ®
© Q5 ©
Each one of the following is associated with papillary necrosis, except:

Acute pyelonephritis

Tuberculosis

Chronic analgesia use

Syphilis

Sickle cell disease

Submit answer

Reference ranges >

Score: 25%

1 X
2 X
3
4 X
5

CamScanner
Acute pyelonephritis

Tuberculosis

Chronic analgesia use

Syphilis

Sickle cell disease

si f # Discuss (2) Improve

Next question >


Papillary necrosis

Causes
• chronic analgesia use
• sickle cell disease
• TB
• acute pyelonephritis
• diabetes mellitus

Features
• fever, loin pain, haematuria
• IVU - papillary necrosis with renal scarring - 'cup & spill

Next question >


CamScanner
Q Q7 ©
A 33-year-old man with a history of coeliac disease is admitted for
investigation of recurrent macroscopic haematuria. His urine is typically
brown and there is no history of passing clots. What is the most likely
diagnosis?

Diffuse proliferative glomerulonephritis

IgA nephropathy

Membranous glomerulonephritis

Minimal change disease

Rapidly progressive glomerulonephritis

This man has IgA nephropathy which is associated with coeliac disease

ii f # Discuss Improve

Next question >


IgA nephropathy

IgA nephropathy (also known as Berger 's disease) is the commonest


cause of glomerulonephritis worldwide. It classically presents as
macroscopic haematuria in young people following an upper respiratory
tract infection.

Associated conditions
• alcoholic cirrhosis
CamScanner
Q Q8 C
A 79-year-old complains of lower urinary tract symptoms. Which one of
the following statements regarding benign prostatic hyperplasia is
incorrect?

Goserelin is licensed for refractory cases

Side-effects of 5 alpha-reductase inhibitors include ejaculation


disorders and gynaecomastia

Possible presentations include recurrent urinary tract infection

5 alpha-reductase inhibitors typically decrease the prostate


specific antigen level

More common in black men

Submit answer

Reference ranges v

Score: 42.9%

1 X
2 X
3
4 X
5 X

CamScanner
Goserelin is licensed for refractory cases

Side-effects of 5 alpha-reductase inhibitors include ejaculation


disorders and gynaecomastia

Possible presentations include recurrent urinary tract infection

5 alpha- reductase inhibitors typically decrease the prostate specific


antigen level

More common in black men

Goserelin (Zoladex) is not used in the management of benign prostatic


hyperplasia

it f # Discuss (1 ) Improve

Next question >

Benign prostatic hyperplasia

Benign prostatic hyperplasia (BPH) is a common condition seen in older


men.

Risk factors
• age: around 50% of 50-year-old men will have evidence of BPH
and 30% will have symptoms . Around 80% of 80- year-old men
have evidence of BPH
• ethnicity: black > white > Asian

BPH typically presents with lower urinary tract symptoms (LUTS), which
may be categorised into:
• voiding symptoms (obstructive): weak or intermittent urinary flow,
straining, hesitancy, terminal dribbling and incomplete emptying
CamScanner
BPH typically presents with lower urinary tract symptoms (LUTS), which
may be categorised into:
• voiding symptoms (obstructive): weak or intermittent urinary flow,
straining, hesitancy, terminal dribbling and incomplete emptying
• storage symptoms (irritative) urgency, frequency, urgency
incontinence and nocturia
• post-micturition: dribbling
• complications: urinary tract infection, retention, obstructive
uropathy

Management options
• watchful waiting
• medication: alpha-1 antagonists, 5 alpha-reductase inhibitors. The
use of combination therapy was supported by the Medical
Therapy Of Prostatic Symptoms (MTOPS) trial
• surgery: transurethral resection of prostate (TURP)

Alpha-1 antagonists e.g. tamsulosin, alfuzosin


• decrease smooth muscle tone (prostate and bladder)
• considered first-line, improve symptoms in around 70% of men
• adverse effects: dizziness, postural hypotension, dry mouth,
depression

5 alpha -reductase inhibitors e.g. finasteride


• block the conversion of testosterone to dihydrotestosterone
(DHT), which is known to induce BPH
• unlike alpha-1 antagonists causes a reduction in prostate volume
and hence may slow disease progression. This however takes
time and symptoms may not improve for 6 months. They may also
decrease PSA concentrations by up to 50%
• adverse effects: erectile dysfunction, reduced libido, ejaculation
problems, gynaecomastia

Next question >


CamScanner
Q9 p

You are prescribing maintenance fluids for a 60-year -old woman who
has had a stroke. Her most recent blood results are shown below:

Na + 140 mmol/I

K+ 4.0 mmol/I

Urea 5.0 mmol/I

Creatinine 88 pmol /l

She weighs 62 kg. What is the most appropriate amount of potassium


that she should receive over a 24-hour period?

20 mmol

40 mmol

60 mmol

80 mmol

120 mmol

Submit answer

Reference ranges v

Score: 50%
CamScanner
60 mmol

80 mmol

120 mmol

When prescribing fluids, the potassium requirement per day is 1


mmol/ kg / day
Importance: 56

Converting this into real- world practice, NICE also recommends that
patients have 25-30ml/ kg/ day of water. As this patient weighs 62kg that
means the patient should receive approximately 1.5 - 2L of fluid in a 24
hour period. If the patient is receiving 1-litre bags of fluid then one
should contain 40 mmol and the other 20 mmol potassium to ensure an
adequate potassium intake.

A # Discuss (1) Improve

Next question >


Fluid therapy

The prescription of intravenous fluids is one of the most common tasks


that junior doctors need to do.

In the 2013 guidelines NICE recommend the following requirements for


maintenance fluids:
• 25-30 ml/kg/ day of water and
• approximately ! mmol/kg/ day of potassium, sodium and chloride
and
• approximately 50-100 g/ day of glucose to limit starvation ketosis

CamScanner
Fluid therapy

The prescription of intravenous fluids is one of the most common tasks


that junior doctors need to do.

In the 2013 guidelines NICE recommend the following requirements for


maintenance fluids:
• 25-30 mi/ kg/ day of water and
• approximately ! mmol/ kg/ day of potassium, sodium and chloride
and
• approximately 50-100 g/ day of glucose to limit starvation ketosis

So, for a 80kg patient, for a 24 hour period, this would translate to:
• 2 litres of water
• 80mmol potassium

For the first 24 hours NICE recommend the following::

When prescribing for routine maintenance alone, consider using


25-30 mi / kg / day sodium chloride 0.18 % in 4 % glucose with 27
mmol / I potassium on day 7 (there are other regimens to achieve
this ).

The amount of fluid patients require obviously varies according to their


recent and past medical history. For example a patient who is post -op
and is having significant losses from drains will require more fluid
whereas a patient with heart failure should be given less fluid to avoid
precipitating pulmonary oedema.

The table below shows the electrolyte concentrations (in


millimoles/ litre) of plasma and the most commonly used fluids:

CamScanner
and is having significant losses from drains will require more fluid
whereas a patient with heart failure should be given less fluid to avoid
precipitating pulmonary oedema .

The table below shows the electrolyte concentrations (in


millimoles/ litre) of plasma and the most commonly used fluids:

Na+ Cl’
K+ HC03- Glucose

Plasma 135- 98- 3.5- 22-28


145 105 5

0.9 % saline 154 154

5 % glucose 50 g

0.18% saline with 30 30 40g


4% glucose

Hartmann's solution 131 111 5 29

Specific points

0.9% saline
• if large volumes are used there is an increased risk of
hyperchloraemic metabolic acidosis

Hartmann's
• contains potassium and therefore should not be used in patients
with hyperkalaemia

Next question >


CamScanner
© Q10 p ©
A 24-year-old man who has a sister with adult polycystic kidney disease
(ADPKD) asks if he could be screened for the disease. What is the most
appropriate screening test ?

PKD1 gene testing

CT abdomen

Urine microscopy

Ultrasound abdomen

Anti-polycystin 1 antibodies levels

Submit answer

Reference ranges v

Score: 55.6%

1 X
2 X
3
4 X
5 X
6 v
'
7
CamScanner
Ultrasound abdomen

Anti-polycystin 1 antibodies levels

Ultrasound is the screening test for adult polycystic kidney


disease
Importance: 14

Genetic testing is still not routinely recommended for screening family


members. Sensitivity for ADPKD 1 is 99 % for at-risk patients older than
20 years

at - t # Discuss (2) Improve

Next question >

ADPKD

Autosomal dominant polycystic kidney disease (ADPKD) is the most


common inherited cause of kidney disease, affecting 1 in 1 ,000
Caucasians . Two disease loci have been identified, PKD 1 and PKD 2,
which code for polycystin-1 and polycystin- 2 respectively

ADPKD type 1 ADPKD type 2

85 % of cases 15 % of cases

Chromosome 16 Chromosome 4

Presents with renal failure earlier

The screening investigation for relatives is abdominal ultrasound:

CamScanner
57 Q13 >/ Ia 57
A 33-year-old pregnant woman presents for advice. She is known to
have polycystic kidney disease but is currently well. Her father also has
polycystic kidneys and is on dialysis . What is the chance her child will
also have the disease?

50% if male

50%

25%

0%

100%

Polycystic kidney disease is usually inherited in an autosomal dominant


fashion and hence 50% of her children will be affected, regardless of
gender. The autosomal recessive form is rare and usually causes death
in childhood.

f # Discuss (3) Improve

Next question >

ADPKD

Autosomal dominant polycystic kidney disease (ADPKD) is the most


common inherited cause of kidney disease, affecting 1 in 1,000
Caucasians. Two disease loci have been identified, PKD1 and PKD 2,
which code for polycystin-1 and polycystin-2 respectively

ADPKD type 1 ADPKD type 2

CamScanner
G Q14 PJ ©
Why do patients with chronic kidney disease have a raised phosphate
level?

Decreased renal excretion

Increased gut absorption

Hypervitaminosis D

Primary hyperparathyroidism

Decreased 25-alpha hydroxylation of vitamin D

Submit answer

Reference ranges v

Score: 69.2%

1 X
2 X
3
4 X

CamScanner
Why do patients with chronic kidney disease have a raised phosphate
level ?

Decreased renal excretion

Increased gut absorption

Hypervitaminosis D

Primary hyperparathyroidism

Decreased 25- alpha hydroxylation of vitamin D

t # Discuss (1 ) Improve

Next question >

Chronic kidney disease: bone disease

Basic problems in chronic kidney disease


• low vitamin D (1- alpha hydroxylation normally occurs in the
kidneys)
• high phosphate
• low calcium: due to lack of vitamin D, high phosphate
• secondary hyperparathyroidism: due to low calcium, high
phosphate and low vitamin D

Several clinical manifestations may result:

Osteitis fibrosa cystica


• aka hyperparathyroid bone disease

CamScanner
Several clinical manifestations may result:

Osteitis fibrosa cystica


• aka hyperparathyroid bone disease

Adynamic
• reduction in cellular activity (both osteoblasts and osteoclasts) in
bone
• may be due to over treatment with vitamin D

Osteomalacia
• due to low vitamin D

Osteosclerosis

Osteoporosis

CamScanner
© Q16 p o
Which one of the following statements regarding the assessment of
proteinuria in patients with chronic kidney disease is NOT true?

Albumin:creatinine ratio (ACR) is more sensitive than


protein:creatinine ratio (PCR)

An ACR of 30 mg/ mmol is approximately equal to a PCR of 50


mg/ mmol

An ACR sample is collected over 24 hours

Women typically have higher ACR values

An ACR of 3.8 mg/ mmol in a diabetic man is clinically significant

Submit answer

Reference ranges v

Score: 60%

X
2 X

CamScanner
An ACR sample is collected over 24 hours

Women typically have higher ACR values

An ACR of 3.8 mg/ mmol in a diabetic man is clinically significant

at # Discuss (3) Improve

Next question >


Chronic kidney disease: proteinuria

Proteinuria is an important marker of chronic kidney disease, especially


for diabetic nephropathy. NICE recommend using the albumin:creatinine
ratio (ACR) in preference to the protein:creatinine ratio (PCR) when
identifying patients with proteinuria as it has greater sensitivity. For
quantification and monitoring of proteinuria, PCR can be used as an
alternative, although ACR is recommended in diabetics. Urine reagent
strips are not recommended unless they express the result as an ACR

Approximate equivalent values

ACR PCR Urinary protein excretion


(mg/mmol) (mg/mmol) (g/24 h)

30 50 0.5

70 100 1

Collecting an ACR sample


CamScanner
(mg/ mmol) (mg/mmol) (g7Z4 h)

30 50 0.5

70 100 1

Collecting an ACR sample


• by collecting a 'spot ' sample it avoids the need to collect urine
over a 24 hour period in order to detect or quantify proteinuria
• should be a first -pass morning urine specimen
• if the initial ACR is between 3 mg/ mmol and 70 mg/mmol, this
should be confirmed by a subsequent early morning sample. If the
initial ACR is 70 mg/ mmol or more, a repeat sample need not be
tested.

Interpreting the ACR results


• the NICE guidelines state 'regard a confirmed ACR of 3 mg/ mmol
or more as clinically important proteinuria'

NICE recommendations for referral to a nephrologist:


• a urinary albumin:creatinine ratio (ACR) of 70 mg/ mmol or more,
unless known to be caused by diabetes and already appropriately
treated
• a urinary ACR of 30 mg/ mmol or more, together with persistent
haematuria (two out of three dipstick tests show 1+ or more of
blood) after exclusion of a urinary tract infection
• consider referral to a nephrologist for people with an ACR
between 3-29 mg/ mmol who have persistent haematuria and
other risk factors such as a declining eGFR, or cardiovascular
disease

Frequency of monitoring eGFR (number of times per year by eGFR and


ACR categories) for people with or at risk of CKD

CamScanner
Frequency of monitoring eGFR (number of times per year by eGFR and
ACR categories) for people with or at risk of CKD

ACR
eGFR categories categories
(mL/min/ 1.73 m2) (mg/ mmol)

A1 (< 3) A 2 (3-30) A 3 (> 30)


Normal to Moderately Severely
mildly increased increased
increased

G1 >=90 Normal and =< 1 1 >= 1


high

G 2 60-89 Mild =< 1 1 >= 1


reduction related to
normal range for a
young adult

G 3 a 45- 59 Mild to 1 1 2
moderate reduction

G3 b 30-44 Moderate to =< 2 2 >= 2


severe reduction

G 4 15-29 Severe 2 2 3
reduction

G 5 <15 Kidney failure 4 >= 4 >= 4

Next question >

B / a A E: ^ Tr H" H GO

CamScanner -;
© Q2 P

A 7-year-old boy presents with polyuria and polydipsia.

He is usually fit and well, and takes no regular medications.

Blood tests reveal:

Fasting blood glucose 5.1 mmol / L (3.9-7.1)

Na + 137 mmol/ L (135 - 145)

K+ 3.8 mmol / L (3.5 - 5.0)

Calcium 2.3 mmol/ L (2.1-2.6)

Thyroid stimulating hormone (TSH) 1.0 mU/ L (0.5- 5.5)

Urine dipstick is normal and a water deprivation test is pending.

Given the likely diagnosis, which gene is most likely to be mutated?

APC

GCK

INS

HNF1 A

AVPR 2

Submit answer

CamScanner
AVPR 2

Nephrogenic diabetes insipidus may be caused genetic


mutations:
• the more common form affects the vasopression ( ADH)
receptor
• the less common form results from a mutation in the gene
that encodes the aquaporin 2 channel

Importance: 50

The differential for the combination of polyuria and polydipsia is wide,


including endocrine, renal, infective, congenital and metabolic causes.
The initial blood tests in this scenario rule out common electrolyte
disturbances (such as hypercalcaemia), hypothyroidism and diabetes
mellitus. Equally, there is no indication that underlying medical
problems (e.g. sickle cell nephropathy) or medications, such as
diuretics, are precipitating the symptoms. A water deprivation test can
be used to differentiate diabetes insipidus from psychogenic polydipsia.

For most hereditary cases of nephrogenic diabetes insipidus, the


inheritance is X-linked recessive. Mutations of the AVPR 2 (arginine
vasopressin receptor 2) gene on the X chromosome are most common,
causing a misfolded protein to become trapped within the cell.
Functional vasopressin 2 receptors are needed to interact with
antidiuretic hormone (ADH). As a result of this mutation, the collecting
ducts in the kidneys do not absorb water leading to an inappropriately
high urine output.

Most hereditary forms of the disease are diagnosed in childhood. This


is in contrast to acquired forms, such as cases caused by lithium, which
are commoner in adults and the onset of symptoms may be slower.

APC (adenomatous polyposis coli) tumour suppressor gene mutations


lead to the condition familial adenomatous polyposis (FAP), an
autosomal dominant inherited condition which increases the risk of
colorectal cancer.
CamScanner
colorectal cancer.

INS (insulin gene) has been associated with the development of type 1
diabetes mellitus, where insulin production from beta cells is impaired.

GCK (glucokinase) mutations are linked to one of the types of


monogenic, familial diabetes: MODY (maturity onset diabetes of the
young).

HNF1 A (HNF1 homeobox A) mutations can similarly cause this familial


type of diabetes, MODY 3. This is the most common type of MODY in
Europe, accounting for approximately 70% of all cases.

A # Discuss Improve

Next question >


Diabetes insipidus

Diabetes insipidus (Dl) is a condition characterised by either a


deficiency of antidiuretic hormone, ADH, (cranial Dl) or an insensitivity
to antidiuretic hormone (nephrogenic Dl).

Causes of cranial Dl
• idiopathic
• post head injury
• pituitary surgery
• craniopharyngiomas
• histiocytosis X
• DIDMOAD is the association of cranial Diabetes Insipidus,
Diabetes Mellitus, Optic Atrophy and Deafness (also known as
Wolfram's syndrome)
• haemochromatosis

Causes of nephrogenic Dl
CamScanner
Causes of nephrogenic Dl
• genetic: the more common form affects the vasopression (ADH)
receptor, the less common form results from a mutation in the
gene that encodes the aquaporin 2 channel
• electrolytes: hypercalcaemia, hypokalaemia
• drugs: demeclocycline, lithium
• tubulo-interstitial disease: obstruction, sickle-cell, pyelonephritis

Features
• polyuria
• polydipsia

Investigation
• high plasma osmolality, low urine osmolality
• a urine osmolality of >700 mOsm/kg excludes diabetes insipidus
• water deprivation test

Management
• nephrogenic diabetes insipidus: thiazides, low salt / protein diet
- central diabetes insipidus can be treated with desmopressin

Next question >


B / a A •Hi

=~ Tr H "' E C-D

Save my notes

Search

CamScanner
Q Q3 P O
A 44-year-old man is referred to the renal team. He has a long history of
chronic sinusitis and was investigated last year for haemoptysis but no
cause was found. A number of recent urine dipstick tests has shown
persistent microscopic haematuria.

Na+ 140 mmol/I

K+ 4.8 mmol/l

Urea 11.4 mmol/l

Creatinine 145 pmol/l

ESR 61 mm/hr

CRP 30 mg/I

anti-GBM Negative

cANCA (PR3) Positive

pANCA (MPO) Negative

ANA Negative

Given the likely diagnosis, what findings would be expected on renal


biopsy ?

Segmental tuft necrosis

Kimmelstiel-Wilson nodules

Crescentic glomerulonephritis

'Full-house' immunoglobulin deposition

Membranous glomerulonephritis

CamScanner
Crescentic glomerulonephritis

Full-house' immunoglobulin deposition

Membranous glomerulonephritis

B6 f # Discuss (3) Improve

Next question >


Rapidly progressive glomerulonephritis

Rapidly progressive glomerulonephritis is a term used to describe a


rapid loss of renal function associated with the formation of epithelial
crescents in the majority of glomeruli.

Causes
• Goodpasture's syndrome
• Wegener's granulomatosis
• others: SLE, microscopic polyarteritis

Features
• nephritic syndrome: haematuria with red cell casts, proteinuria,
hypertension, oliguria
• features specific to underlying cause (e.g. haemoptysis with
Goodpasture's, vasculitic rash or sinusitis with Wegener 's)
© Q4 \C3

You are reviewing a 33-year -old man who has recently been diagnosed
with adult polycystic kidney disease in the renal clinic. You proceed to
examine his cardiovascular system. Which other feature are you most
likely to find on examination?

Dilated cardiomyopathy

Mitral stenosis

Aortic stenosis

Renal bruit secondary to renal artery stenosis

Mitral valve prolapse

Submit answer

Reference ranges v

Score: 66.7%

1 <

2 X

CamScanner
G Q4 >/ P ©
You are reviewing a 33- year-old man who has recently been diagnosed
with adult polycystic kidney disease in the renal clinic. You proceed to
examine his cardiovascular system . Which other feature are you most
likely to find on examination?

Dilated cardiomyopathy

Mitral stenosis

Aortic stenosis

Renal bruit secondary to renal artery stenosis

Mitral valve prolapse

ADPKD is associated with mitral valve prolaspe


Important for me Less important

# Discuss (1 ) Improve

Next question >

ADPKD: features

CamScanner
B6 f # Discuss (1) Improve

Next question >


ADPKD: features

Features
• hypertension
• recurrent UTIs
• abdominal pain
• renal stones
• haematuria
• chronic kidney disease

Extra-renal manifestations
• liver cysts (70% - the commonest extra-renal manifestation): may
cause hepatomegaly
• berry aneurysms (8%): rupture can cause subarachnoid
haemorrhage
• cardiovascular system: mitral valve prolapse, mitral/tricuspid
incompetence, aortic root dilation, aortic dissection
• cysts in other organs: pancreas, spleen; very rarely: thyroid,
oesophagus, ovary
© Q5 p ©
A 54-year-old truck driver presented to emergency department with a
new onset severe back pain radiating to his groin. He required 10 mg of
intravenous morphine to control his pain. He is not a smoker but drinks
10-12 pints on weekends. He has recently been diagnosed with
esophageal cancer and has undertaken his first chemotherapy session
the previous week . With regards to uric acid calculi, one of its
remarkable characteristic is:

They cannot be visualised by ultrasonography (US)

They are radiolucent

Hardly seen on plain CT (CT KUB)

High solubility on acidic urine pH

Poor response to medical treatment

Submit answer

Reference ranges v

Score: 75%

1 v
'
2 X
3 s/
CamScanner
They are radiolucent

Hardly seen on plain CT (CT KUB)

High solubility on acidic urine pH

Poor response to medical treatment

Uric acid nephrolithiasis are radiolucent, requiring


ultrasonography or CT KUB (without contrast)
Importance: 65

Uric acid nephrolithiasis are radiolucent, requiring ultrasonography or


CT KUB ( without contrast) instead of a plain film. With ultrasonography
can demonstrate a calculi by an echogenic foci with distal acoustic
shadowing regardless of its nature. They respond well to medical
treatment which includes, hydration, urinary alkalinization and
allopurinol if raised uric acid levels. Uricosuric medications should be
avoided, especially in renal impairment, as it can precipitate the
formation of new calculi.

# Discuss (1) Improve

Next question >


Renal stones: imaging

The table below summarises the appearance of different types of renal


stone on x-ray

Radiograph
Type Frequency appearance

P aIo i l i m aI o t o /in% Ononi 10


CamScanner
.MUIIC uii A ' i a y

Radiograph
Type Frequency appearance

Calcium oxalate 40% Opaque

Mixed calcium 25% Opaque


oxalate/ phosphate stones

Triple phosphate stones* 10% Opaque

Calcium phosphate 10% Opaque

Urate stones 5-10% Radio-lucent

Cystine stones 1% Semi-opaque, 'ground-


glass ' appearance

Xanthine stones <1 % Radio-lucent

*stag-horn calculi involve the renal pelvis and extend into at least 2
calyces. They develop in alkaline urine and are composed of struvite
(ammonium magnesium phosphate, triple phosphate). Ureaplasma
urealyticum and Proteus infections predispose to their formation

Next question >

B / a A H
E: ~ TIT H" s GO

Save my notes

CamScanner
© Q7

A 71-year-old man with chronic kidney disease stage 3 is reviewed in


the cardiology clinic. He is known to have hypertension and ischaemic
heart disease but a recent fasting glucose result confirmed he is not
diabetic . A recent early morning urine result is reported as follows:

1 week ago 6 months ago

Na + 141 mmol / l 140 mmol/ l

K+ 4.1 mmol /l 4.3 mmol / l

Urea 7.2 mmol / l 6.3 mmol / l

Creatinine 98 (jmol/ l 99 (jmol/ l

1 week ago 6 months ago

Albumin:creatinine ratio 15.2 mg / mmol 2.6 mg / mmol

What is the most appropriate action?

Refer to a nephrologist

Repeat the sample

Obtain a 24-hour urine collection

Repeat using a late-evening sample

Arrange renovascular imaging

Submit answer

CamScanner
Repeat the sample

Obtain a 24-hour urine collection

Repeat using a late-evening sample

Arrange renovascular imaging

NICE recommend 'if the initial ACR is between 3 mg/ mmol and 70
mg/ mmol, this should be confirmed by a subsequent early morning
sample. If the initial ACR is 70 mg/ mmol or more, a repeat sample need
not be tested.'

A ? # Discuss (2) Improve

Next question >


Chronic kidney disease: proteinuria

Proteinuria is an important marker of chronic kidney disease, especially


for diabetic nephropathy. NICE recommend using the albumin:creatinine
ratio ( ACR) in preference to the protein:creatinine ratio (PCR) when
identifying patients with proteinuria as it has greater sensitivity. For
quantification and monitoring of proteinuria, PCR can be used as an
alternative, although ACR is recommended in diabetics. Urine reagent
strips are not recommended unless they express the result as an ACR

Approximate equivalent values

ACR PCR Urinary protein excretion


(mg/mmol) (mg/mmol) (g/ 24 h)

30 50 0.5

CamScanner
Q8 p

Which one of the following is associated with a better prognosis in


patients with IgA nephropathy ?

Heavy proteinuria at presentation

Male gender

Hyperlipidaemia

Frank haematuria

ACE genotype DD

Submit answer

Reference ranges v

Score: 85.7%

1 >/

2 X
3 >/

4
5
'V
v

6 >/

7
Q
CamScanner
© Q8 p ©
Which one of the following is associated with a better prognosis in
patients with IgA nephropathy ?

Heavy proteinuria at presentation

Male gender

Hyperlipidaemia

Frank haematuria

ACE genotype DD

f 0 Discuss (1) Improve

Next question >


IgA nephropathy

IgA nephropathy (also known as Berger 's disease) is the commonest


cause of glomerulonephritis worldwide. It classically presents as
macroscopic haematuria in young people following an upper respiratory
tract infection.

Associated conditions
• alcoholic cirrhosis
• coeliac disease / dermatitis herpetiformis
• Hpnnrh-^rhnnlpin ni irm ira
.
CamScanner
G Q1 P

A 6-year -old male is brought to the emergency department by his


mother. He presents with a 5-day history of high fever, dysuria and
lethargy. The mother states she is encouraging him to drink water but
despite this, he is urinating small amounts.

Urine dip is positive for leukocytes, nitrates and the pH level is alkaline.

The sample is further sent for microscopy, culture and sensitivity, which
revealed numerous gram-negative motile rod- shaped bacteria indicating
Proteus mirabilis .

Which of the following types of renal stones does this infection


increase the risk of developing?

Cysteine stone

Struvite stone

Calcium oxalate stone

Uric acid stone

Xanthine stone

Submit answer

CamScanner
Struvite stone

Calcium oxalate stone

Uric acid stone

Xanthine stone

Proteus mirabilis infection predisposes to struvite kidney stones


Important for me Less important

P mirabilis is a gram negative, facultatively anaerobic, rod-shaped


bacterium which is motile. The likely cause of a patient presenting with
alkaline urine is P mirabilis . This organism produces urease which
breaks down urea to carbon dioxide and ammonia resulting in a lower
pH. The alkaline urine increases the risk of developing struvite stones.

Calcium oxalate stones are the most common types of stones in adults.
Low calcium diets and dehydration increase the risk of the development
of these stones.

Uric acid stones are the most common stones seen in patients with
gout. Dehydration and having acidic urine increase the risk of
developing these stones.

Cysteine stones tend to be related to a genetic defect resulting in a


decreased reabsorption of dibasic amino acids which includes cysteine.

Xanthine stones are rare. Genetic defects altering xanthine oxidase


result in the inability to convert xanthine to uric acid.

A f # Discuss (1) Improve

Next question >


CamScanner
of these stones .

Uric acid stones are the most common stones seen in patients with
gout . Dehydration and having acidic urine increase the risk of
developing these stones .

Cysteine stones tend to be related to a genetic defect resulting in a


decreased reabsorption of dibasic amino acids which includes cysteine.

Xanthine stones are rare. Genetic defects altering xanthine oxidase


result in the inability to convert xanthine to uric acid .

>4 # Discuss (1 ) Improve

Next question >

Xanthogranulomatous pyelonephritis

Xanthogranulomatous pyelonephritis is a rare chronic granulomatous


disease resulting in a non- functioning kidney. Chronic/ subacute
infection by organisms such as Proteus mirabilis predispose to renal
stones including staghorn calculi. Foamy (lipid laden) macrophages are
typically seen .

Next question >

B / B A •B
= w
Tr E co

Save my notes

Search

CamScanner
© Q3 p c
A 40-year-old man with a history of psychiatric problems and epilepsy
comes for review. He complains that he is drinking excessive amounts
of water and having to urinate frequently. He has not lost any weight
and states that he is compliant with his current medications. Blood
tests show the following:

Na+ 145 mmol/l

K+ 4.1 mmol/l

Urea 6.3 mmol/l

Creatinine 101 pmol/ l

Glucose (random) 6.2 mol/ l

Which one of the following medications is most likely to be responsible


for this presentation?

Carbamazepine

Fluoxetine

Olanzapine

Sodium valproate

Lithium

Submit answer

CamScanner
Lithium

insipidus
This patient has probably developed nephrogenic diabetes
secondary to lithium therapy. Polyuria, polydipsia and a
high-normal

sodium are pointers towards this.

f 0 Discuss (2) Improve


i
*
Next question >

Diabetes insipidus

Diabetes insipidus (Dl) is a condition characterised by either a


deficiency of antidiuretic hormone, ADH, (cranial Dl) or an insensitivity
to antidiuretic hormone (nephrogenic Dl).

Causes of cranial Dl
• idiopathic
• post head injury
• pituitary surgery
• craniopharyngiomas
• histiocytosis X
• DIDMOAD is the association of cranial Diabetes Insipidus,
Diabetes Mellitus, Optic Atrophy and Deafness (also known as
Wolfram's syndrome)
• haemochromatosis

Causes of nephrogenic Dl
• genetic: the more common form affects the vasopression (ADH)
receptor, the less common form results from a mutation in the
gene that encodes the aquaporin 2 channel
• electrolytes: hypercalcaemia, hypokalaemia
• drugs: demeclocycline, lithium
• tubulo-interstitial disease: obstruction, sickle-cell, pyelonephritis
CamScanner -;
, upuu m i u p n y «5
^o u u
uiaueuij ivieiiuuj cniu UCBIIICJJ MIUVVII

Wolfram's syndrome)
• haemochromatosis

Causes of nephrogenic Dl
• genetic: the more common form affects the vasopression (ADH)
receptor, the less common form results from a mutation in the
gene that encodes the aquaporin 2 channel
• electrolytes: hypercalcaemia, hypokalaemia
• drugs: demeclocycline, lithium
• tubulo-interstitial disease: obstruction, sickle-cell, pyelonephritis

Features
• polyuria
• polydipsia

Investigation
• high plasma osmolality, low urine osmolality
• a urine osmolality of >700 mOsm /kg excludes diabetes insipidus
• water deprivation test

Management
• nephrogenic diabetes insipidus: thiazides, low salt / protein diet
- central diabetes insipidus can be treated with desmopressin

Next question >

B / a A =' TI" H C-D

Save my notes

CamScanner
o Q4 X p ©
A 62-year -old man attends your clinic. He has a history of hypertension
and atrial fibrillation for which he is anticoagulated with warfarin. A
urine dipstick taken 8 weeks ago during a routine hypertension clinic
appointment showed blood + with leucocytes +. Initial urine microscopy
and culture shows no growth. The urine dipstick has been repeated on
two further occasions with the same finding.

What is the most appropriate action?

Take no further action

Send a 24-urine sample for protein estimation

Refer to nephrology

Refer to urology

Send a further urine microscopy

The incidence of non-visible haematuria is similar in patients taking


warfarin to the general population, therefore, these patients should be
investigated as normal.

Most haematuria protocols suggest sending younger patients (e. g. < 40


years) to nephrology initially but as this patient is older he should be
sent to urology for a cystoscopy.

4 f # Discuss Improve

CamScanner
Haematuria

The management of patients with haematuria is often difficult due to


the absence of widely followed guidelines. It is sometimes unclear
whether patients are best managed in primary care, by urologists or by
nephrologists.

The terminology surrounding haematuria is changing. Microscopic or


dipstick positive haematuria is increasingly termed non-visible
haematuria whilst macroscopic haematuria is termed visible
haematuria. Non-visible haematuria is found in around 2.5% of the
population.

Causes of transient or spurious non-visible haematuria


• urinary tract infection
• menstruation
• vigorous exercise (this normally settles after around 3 days)
• sexual intercourse

Causes of persistent non-visible haematuria


• cancer (bladder, renal, prostate)
• stones
• benign prostatic hyperplasia
• prostatitis
• urethritis e.g. Chlamydia
• renal causes: IgA nephropathy, thin basement membrane disease

Spurious causes - red/ orange urine, where blood is not present on


dipstick
• foods: beetroot, rhubarb
• drugs: rifampicin, doxorubicin

Management

Current evidence does not support screening for haematuria. The


CamScanner
Management

Current evidence does not support screening for


haematuria. The
taking
incidence of non-visible haematuria is similar in patients
aspirin/ warfarin to the general population hence these
patients should

also be investigated.

Testing
• urine dipstick is the test of choice for detecting haematuria
being
• persistent non-visible haematuria is often defined as blood
present in 2 out of 3 samples tested 2-3 weeks apart
• renal function, albumin:creatinine ( ACR) or protein:creatinine ratio
(PCR) and blood pressure should also be checked
• urine microscopy may be used but time to analysis significantly
affects the number of red blood cells detected

NICE urgent cancer referral guidelines were updated in 2015.

Urgent referral (i. e. within 2 weeks)

Aged > = 45 years AND:


• unexplained visible haematuria without urinary tract infection, or
• visible haematuria that persists or recurs after successful
treatment of urinary tract infection

Aged >= 60 years AND have unexplained nonvisible haematuria and


either dysuria or a raised white cell count on a blood test

Non- urgent referral

Aged 60 >= 60 years with recurrent or persistent unexplained urinary


tract infection

Since the investigation (or not) of non-visible haematuria is such as a


common dilemma a number of guidelines have been published. They
generally agree with NICE guidance, of note:
^ -=
CamScanner -; L 3
(PCR) and blood pressure should also be checked
• urine microscopy may be used but time to analysis significantly
affects the number of red blood cells detected

NICE urgent cancer referral guidelines were updated in 2015.

Urgent referral (i.e. within 2 weeks)

Aged >= 45 years AND:


• unexplained visible haematuria without urinary tract infection, or
• visible haematuria that persists or recurs after successful
treatment of urinary tract infection

Aged >= 60 years AND have unexplained nonvisible haematuria and


either dysuria or a raised white cell count on a blood test

Non- urgent referral

Aged 60 >= 60 years with recurrent or persistent unexplained urinary


tract infection

Since the investigation (or not) of non-visible haematuria is such as a


common dilemma a number of guidelines have been published. They
generally agree with NICE guidance, of note:
• patients under the age of 40 years with normal renal function, no
proteinuria and who are normotensive do not need to be referred
and may be managed in primary care

Next question >


B / a A HI =~ Tr SS " E C-D

Save my notes
CamScanner
G Q6 P ©
Which one of the following is the most common type of SLE associated
renal disease?

Class II: mesangial glomerulonephritis

Class III: focal (and segmental) proliferative glomerulonephritis

Class IV: diffuse proliferative glomerulonephritis

Class V: diffuse membranous glomerulonephritis

Class VI: sclerosing glomerulonephritis

Submit answer

Reference ranges v

Score: 60%

1
2 X
3 V
4 X

CamScanner
© Q6 >/ ©
Which one of the following is the most common type of SLE associated
renal disease?

Class II: mesangial glomerulonephritis

Class III: focal (and segmental) proliferative glomerulonephritis

Class IV: diffuse proliferative glomerulonephritis

Class V: diffuse membranous glomerulonephritis

Class VI: sclerosing glomerulonephritis

Diffuse proliferative glomerulonephritis is the most common and


severe form of renal disease in SLE patients
Importance: 87

at t # Discuss Improve

Systemic lupus erythematosus: renal


complications

Lupus nephritis is a severe manifestation of systemic lupus


erythematosus (SLE) that can result in end-stage renal disease. SLE
patients should be monitored by performing urinalysis at regular check-
up appointments to rule out proteinuria.

CamScanner
Systemic lupus erythematosus: renal
complications

Lupus nephritis is a severe manifestation of systemic lupus


erythematosus (SLE) that can result in end-stage renal disease. SLE
patients should be monitored by performing urinalysis at regular check-
up appointments to rule out proteinuria.

WHO classification
• class I: normal kidney
• class II: mesangial glomerulonephritis
• class III: focal (and segmental) proliferative glomerulonephritis
• class IV: diffuse proliferative glomerulonephritis
• class V: diffuse membranous glomerulonephritis
• class VI: sclerosing glomerulonephritis

Class IV (diffuse proliferative glomerulonephritis) is the most common


and severe form. Renal biopsy characteristically shows the following
findings:
• glomeruli shows endothelial and mesangial proliferation, ' wire-
loop' appearance
• if severe, the capillary wall may be thickened secondary to
immune complex deposition
• electron microscopy shows subendothelial immune complex
deposits
• granular appearance on immunofluorescence

CamScanner
deposits
• granular appearance on immunofluorescence

© Image used on license from PathoPic i


\
Diffuse proliferative SLE. Proliferation of endothelial and
mesangial cells. The thickening of the capillary wall results in a
' wire-loop' appearance. Some crescents are present.

Management
• treat hypertension
• corticosteroids if clinical evidence of disease
• immunosuppressants e.g. azathioprine/ cyclophosphamide

B / a A =1 ^ TIT E GO

Save my notes

Search

Sparr.h tpvthnnk
CamScanner
G Q1 P ©
A 22-year -old man presents to his GP with polyuria, nocturia and
polydipsia. He has no past medical history of note. Routine blood tests
including Hbalc and random glucose are unremarkable.

He undergoes a water deprivation test, which confirms diabetes


insipidus. There is no response to nasal desmopressin.

Which of the following are a cause of this condition?

Craniopharyngioma

Haemochromatosis

Hypocalcaemia

Inherited mutation in the vasopressin ( ADH) receptor

Inherited mutation in the aquaporin 1 channel

Submit answer

Reference ranges v

A O/

CamScanner
Inherited mutation in the vasopressin (ADH) receptor

Inherited mutation in the aquaporin 1 channel

Nephrogenic diabetes insipidus may be caused genetic


mutations:
• the more common form affects the vasopression (ADH)
receptor
• the less common form results from a mutation in the gene
that encodes the aquaporin 2 channel

Important for me Less important

This patient has presented with polyuria and polydipsia; the key
symptoms of diabetes insipidus. The water deprivation test is used to
confirm the diagnosis, and demonstrates an inability to concentrate
urine in affected individuals.

Diabetes insipidus (Dl) is caused by either a deficiency of antidiuretic


hormone (cranial Dl) or an insensitivity to antidiuretic hormone
(nephrogenic Dl). The two may be distinguished by administering nasal
desmopressin - a synthetic form of antidiuretic hormone. Antidiuretic
hormone is also known as vasopressin.

In cases of cranial Dl, there will be a response to the desmopressin, and


urine will become more concentrated following administration.
However, in nephrogenic Dl, as the kidneys are insensitive to antidiuretic
hormone, there will not be a response. The diagnosis in this case is
therefore nephrogenic Dl.

Nephrogenic diabetes insipidus may be caused by genetic mutations:


• The more common form affects the vasopressin (ADH) receptor.
• The less common form results from a mutation in the gene that
encodes the aquaporin 2 channel.

CamScanner
Hypercalcaemia is a cause of nephrogenic diabetes insipidus.

Craniopharyngioma and haemochromatosis are causes of cranial Dl.

A "f # Discuss Improve

Next question >


Diabetes insipidus

Diabetes insipidus (Dl) is a condition characterised by either a


deficiency of antidiuretic hormone, ADH, (cranial Dl) or an insensitivity
to antidiuretic hormone (nephrogenic Dl).

Causes of cranial Dl
• idiopathic
• post head injury
• pituitary surgery
• craniopharyngiomas
• histiocytosis X
• DIDMOAD is the association of cranial Diabetes Insipidus,
Diabetes Mellitus, Optic Atrophy and Deafness (also known as
Wolfram's syndrome)
• haemochromatosis

Causes of nephrogenic Dl
• genetic: the more common form affects the vasopression (ADH)
receptor, the less common form results from a mutation in the
gene that encodes the aquaporin 2 channel
• electrolytes: hypercalcaemia, hypokalaemia
• drugs: demeclocycline, lithium
• tubulo-interstitial disease: obstruction, sickle-cell, pyelonephritis

CamScanner
Causes of nephrogenic Dl
• genetic: the more common form affects the vasopression (ADH)
receptor, the less common form results from a mutation in the
gene that encodes the aquaporin 2 channel
• electrolytes: hypercalcaemia, hypokalaemia
• drugs: demeclocycline, lithium
• tubulo-interstitial disease: obstruction, sickle-cell, pyelonephritis

Features
• polyuria
• polydipsia

Investigation
• high plasma osmolality, low urine osmolality
• a urine osmolality of >700 mOsm/ kg excludes diabetes insipidus
• water deprivation test

Management
• nephrogenic diabetes insipidus: thiazides, low salt / protein diet
- central diabetes insipidus can be treated with desmopressin

Next question >


B / a A =' Tr Mw E c-o

Save my notes

Search

CamScanner
© Q2 p ©
A 23 -year -old patient presents to the emergency department with a
head injury following a night out with his friends. He reports he drank
excess alcohol. His observations are within normal range, pupils equal
and reactive to light. CT head is normal. He complains of feeling thirsty
and needed to urinate excessively.

What is the mechanism of his polyuria ?

Ethanol substrates leads to osmotic diuresis

Inhibition of posterior pituitary gland

Stimulation of posterior pituitary gland

The sugar in alcoholic drinks leads to osmotic diuresis

Ethanol increases the number of aquaporin 2 channels

Alcohol bingeing can lead to ADH suppression in the posterior


pituitary gland subsequently leading to polyuria
Important for me Less important

Dehydration causes increased plasma osmolality which stimulates


vasopressin (anti-diuretic hormone) release from posterior pituitary
gland. This increases insertion of aquaporin 2 channels in the distal
convoluted tubules and collecting duct in the kidney. AQP-2 channels
increase water reabsorption which leads to decreased plasma
osmolality and therefore decreasing the volume of urine produced i.e.
antidiuretic.

Alcohol inhibits this mechanism and subsequently leads to polyuria and


dehydration. Polyuria then leads to the feeling of thirst i.e. polydipsia .

CamScanner
at f # Discuss Improve

Polyuria

A recent review in the BMJ categorised the causes of polyuria by how


common they were. This does not of course tally with how common
they are in exams!

Common ( >1 in 10)


• diuretics, caffeine & alcohol
• diabetes mellitus
• lithium
• heart failure

Infrequent (1 in 100)
• hypercalcaemia
• hyperthyroidism

Rare (1 in 1000)
• chronic renal failure
• primary polydipsia
• hypokalaemia

Very rare (<1 in 10 000)


• diabetes insipidus

Jakes A, Bhandari S. Investigating polyuria . BMJ 2013;347:f 6772.

E I S A •
Hi
=P TP HP E C-D

CamScanner
G Q1 P ©
A 34-year -old single mum has been referred for genetic testing
following her 8-months-old son being diagnosed with congenital
nephrogenic diabetes insipidus. She is asymptomatic and is not aware
of any family history of this condition.

Which of the following structure of the kidney is most commonly


affected in this condition?

Vasopressin receptor

Aquaporin 2 channel (AQP2)

Epithelial sodium channel (ENaC)

Sodium-chloride co-transporter (NCCT)

Aquaporin 1 channel ( AQP1)

Submit answer

Reference ranges v

Score: 0%

CamScanner
Vasopressin receptor

Aquaporin 2 channel (AQP 2)

Epithelial sodium channel (ENaC)

Sodium-chloride co-transporter (NCCT)

Aquaporin 1 channel (AQP1)

Nephrogenic diabetes insipidus may be caused genetic


mutations:
• the more common form affects the vasopression (ADH)
receptor
• the less common form results from a mutation in the gene
that encodes the aquaporin 2 channel

Importance: 50

The correct answer is ' vasopressin receptor '. Nephrogenic diabetes


insipidus is a condition characterised by insensitivity to antidiuretic
hormone ( ADH, aka vasopressin). Genetic is among one of the causes
of this condition and can be due to the following mutation:

• X-linked mutation in the vasopressin (ADH) receptor gene (the


more common form)
• Autosomal recessive defect in the aquaporin 2 gene (the less
common form)

Aquaporin 1 channel is not regulated by vasopressin ( ADH) and is not


associated with nephrogenic diabetes insipidus.

Epithelial sodium channel (ENaC) dysregulation is indicated in Liddle's


syndrome.

CamScanner
Sodium-chloride co-transporter (NCCT) dysfunction is indicated in
Gitelman syndrome.

si ? 0 Discuss Improve

Next question >


Diabetes insipidus

Diabetes insipidus (Dl) is a condition characterised by either a


deficiency of antidiuretic hormone, ADH, (cranial Dl) or an insensitivity
to antidiuretic hormone (nephrogenic Dl).

Causes of cranial Dl
• idiopathic
• post head injury
• pituitary surgery
• craniopharyngiomas
• histiocytosis X
• DIDMOAD is the association of cranial Diabetes Insipidus,
Diabetes Mellitus, Optic Atrophy and Deafness (also known as
Wolfram's syndrome)
• haemochromatosis

Causes of nephrogenic Dl
• genetic: the more common form affects the vasopression (ADH)
receptor, the less common form results from a mutation in the
gene that encodes the aquaporin 2 channel
• electrolytes: hypercalcaemia, hypokalaemia
• drugs: demeclocycline, lithium
• tubulo-interstitial disease: obstruction, sickle-cell, pyelonephritis
CamScanner
Features
• polyuria
• polydipsia

Investigation
• high plasma osmolality, low urine osmolality
• a urine osmolality of >700 mOsm / kg excludes diabetes insipidus
• water deprivation test

Management
• nephrogenic diabetes insipidus: thiazides, low salt / protein diet
- central diabetes insipidus can be treated with desmopressin

Next question >

E I S A a MM
«^M =~ TI" E c-o

Save my notes

Search

Search textbook...

Q Google search on "Diabetes insipidus"

CamScanner
A 38-year- old male with a background of adult polycystic kidney disease
and a preemptive renal transplant 8 years ago presents to the transplant
clinic with worsening bilateral hand tremor. His renal function remains
stable with no uraemia and denies any recent excessive alcohol intake.

What is the most likely cause for his hand tremor ?

Alcohol

Anxiety

Drug-induced parkinsonism

Essential tremor

Tacrolimus

Submit answer

Reference ranges v

Score: 0%

CamScanner
Tacrolimus

Tremor is a common tacrolimus-associated side effect


Importance: 50

This patient is suffering with tremors caused by tacrolimus which is his


long term immunosuppression to maintain the preemptive renal
transplant which he received due to his underlying adult polycystic
kidney disease. Tremor is a common tacrolimus-associated side effect
noticeable with outstretched hands.

Alcohol withdrawal can result in tremors / uncontrolled shaking as can


chronic alcohol consumption which can have a toxic effect on parts of
the brain such as the cerebellum resulting in an intention tremor but is
unlikely in this case given the lack of alcohol intake.

Anxiety can result in involuntary movements as you deal with underlying


physical or emotional stress and worries producing trembling
sensations or even twitching / shaking but there is no evidence of this
within the history.

Drug-induced parkinsonism occurs in patients taking drugs which may


block the action of dopamine but the patient is unlikely to be on such a
medication.

Essential tremor is a progressive disorder of unknown cause resulting


in rhythmic trembling of the hands.

4 f # Discuss Improve

CamScanner
Renal transplant: immunosuppression
Example regime
• initial: ciclosporin/ tacrolimus with a monoclonal antibody
• maintenance: ciclosporin/ tacrolimus with MMF or sirolimus
• add steroids if more than one steroid responsive acute rejection episode

Ciclosporin
• inhibits calcineurin, a phosphotase involved in T cell activation

Tacrolimus
• lower incidence of acute rejection compared to ciclosporin
• also less hypertension and hyperlipidaemia
• however, high incidence of impaired glucose tolerance and diabetes

Mycophenolate mofetil (MMF)


• blocks purine synthesis by inhibition of IMPDH
• therefore inhibits proliferation of B and T cells
• side-effects: Gl and marrow suppression

Sirolimus (rapamycin)
• blocks T cell proliferation by blocking the IL-2 receptor
• can cause hyperlipidaemia

Monoclonal antibodies
• selective inhibitors of IL-2 receptor
• daclizumab
• basilximab

CamScanner
Monitoring

Patients on long-term immunosuppression for organ transplantation require


regular monitoring for complications such as:

Cardiovascular disease - tacrolimus and ciclosporin can cause hypertension and


hyperglycaemia . Tacrolimus can also cause hyperlipidaemia. Patients must be
monitored for accelerated cardiovascular disease.

Renal failure - due to nephrotoxic effects of tacrolimus and ciclosporin/ graft


rejection/ recurrence of original disease in transplanted kidney

Malignancy - patients should be educated about minimising sun exposure to


reduce the risk of squamous cell carcinomas and basal cell carcinomas

B / B A - •
m
wmm
MB
M B
^ E c-D x

Save my notes

M
4
.
» «>

«4
* -- *
to .. fc m>
-*
m 4 .
4 fe

4 4 r
- •<
i« (

•• •»»•»
. -^H* ** «•
4
- - •-
4 %

44*«
#
*
• *

. -
r4M 4

* ••
M

.
4 r
^

— « *, 4
to

tM
-. >«» A
•* ** / 4«
*•. «4 - -
w a 4

Grafting, Organ Transplants and Immunosuppression

CamScanner J ^£> -^11


Q Q1 \C3 ©
A 32-year-old man has been admitted to the endocrinology ward with
polyuria and polydipsia. He feels well otherwise.

He has told your consultant that his brother and maternal uncle have
similar problems but he is the first to seek advice.

Your consultant has arranged investigations.

Na+ 157 mmol/ L (135 - 145)

Glucose 5.9 mmol/ L (4 - 7)

Urine osmolality 178 mOsm / Kg (>300)

Serum osmolality 924 mOsm / Kg (280 - 290)

MRI brain No abnormality

Renal ultrasound Normal appearance of both kidneys

Gene profile Abnormal AVPR 2 gene on chromosome Xq28

Which of the following is the correct diagnosis?

Latent Autoimmune Diabetes in Adults (LADA)

Maturity Onset Diabetes of the Young (MODY)

Nephrogenic diabetes insipidus

Neurogenic diabetes insipidus

Type 2 diabetes mellitus (T2DM)

CamScanner
IVIUIUI U I I O V/ l ^ l V^ O W I L I I V^ I V I V J L/ i y
^
LyiUUV V/ U I
lljf

Nephrogenic diabetes insipidus

Neurogenic diabetes insipidus

Type 2 diabetes mellitus (T2DM)

Nephrogenic diabetes insipidus may be caused genetic


mutations:
• the more common form affects the vasopression (ADH)
receptor
• the less common form results from a mutation in the gene
that encodes the aquaporin 2 channel

Importance: 50

LADA is a type of type 1 diabetes mellitus with a slower onset and is


therefore diagnosed in adults. In many occasions LADA is initially
misdiagnosed as type 2 diabetes. However, there is little response to
medications used to treat type 2 diabetes mellitus as LADA is due to a
lack of insulin production rather than insulin resistance.
Hyperglycaemia is often found along with low C-peptide levels and islet
cell antibodies.

MODY is a family of inherited forms of diabetes mellitus with a strong


autosomal dominant familial link. There are 14 recognised gene
mutations recognised with MODY1 (HNF4A), MODY 2 (glucokinase),
MODY 3 (HNF1 A) and MODY 5 (HNF1 B) being the most common. These
patients have early onset of diabetes, usually in people < 35 years of
age, with symptoms and signs atypical of type 1 diabetes mellitus and
type 2 diabetes mellitus.

CamScanner
Nephrogenic diabetes insipidus is the correct answer. Patients often
present with polyuria and polydipsia and dehydration is common. With
no damage to the pituitary gland or hypothalamus, MRI brain is usually
normal. Urine osmolality is < 300mOsm / kg in nephrogenic diabetes
insipidus. The most common genetic mutation causing nephrogenic
diabetes insipidus affects the arginine vasopressin V 2 receptor
(AVPR 2).

Neurogenic diabetes insipidus is secondary to reduced ADH secretion.


In the majority of cases it is not known why there is a lack of ADH
secretion. In other patients there may be evidence of posterior pituitary,
hypothalamus or pituitary stalk injury on MRI brain. AVP-NPII (arginine
vasopressin-neurophysin II) is the most well known genetic variant of
neurogenic diabetes insipidus, however this is still extremely rare.

Type 2 diabetes mellitus commonly presents with polyuria and


polydipsia as in the above scenario, however T2DM requires a HbA1c of
>48mmol / mol or fasting glucose of 7.0mmol/ L or more. Given that the
patient in the above scenario has a normal glucose, T 2DM is incorrect.

>4 f # Discuss Improve

Next question >


Diabetes insipidus

Diabetes insipidus (Dl) is a condition characterised by either a


deficiency of antidiuretic hormone, ADH, (cranial Dl) or an insensitivity
to antidiuretic hormone (nephrogenic Dl).

Causes of cranial Dl
• idiopathic
• post head injury
• pituitary surgery
• craniopharyngiomas
• histiocytosis X
CamScanner
Causes of cranial Dl
• idiopathic
• post head injury
• pituitary surgery
• craniopharyngiomas
• histiocytosis X
• DIDMOAD is the association of cranial Diabetes Insipidus,
Diabetes Mellitus, Optic Atrophy and Deafness (also known as
Wolfram's syndrome)
• haemochromatosis

Causes of nephrogenic Dl
• genetic: the more common form affects the vasopression (ADH)
receptor, the less common form results from a mutation in the
gene that encodes the aquaporin 2 channel
• electrolytes: hypercalcaemia, hypokalaemia
• drugs: demeclocycline, lithium
• tubulo-interstitial disease: obstruction, sickle-cell, pyelonephritis

Features
• polyuria
• polydipsia

Investigation
• high plasma osmolality, low urine osmolality
• a urine osmolality of >700 mOsm/kg excludes diabetes insipidus
• water deprivation test

Management
• nephrogenic diabetes insipidus: thiazides, low salt / protein diet
- central diabetes insipidus can be treated with desmopressin

Next question >


CamScanner
a Q2 Ia

A 23-year-old male presents with excessive thirst and urination. He also


complains of nocturia which is affecting his sleep. You arrange for
some initial investigations (results shown below) which lead you to
suspect diabetes insipidus as the diagnosis. You plan to arrange a
water deprivation test to investigate further.

Na+ 144 mmol/ L

K+ 4.2 mmol/L

Urea 6.8 mmol / L

Creatinine 79 pmol/ L

Calcium 2.3 mmol / L

HbA1c 37 mmol/ L

Serum osmolality 305 mOsmol/kg

24-hour urine collection 3.5L

Urine dipstick negative

Urine osmolality 75 mOsmol / kg

Which of the following is a nephrogenic cause of the suspected


diagnosis?

Idiopathic

Head injury

Haemochromatosis

Mutations in the gene encoding the vasopressin receptor

Wolfram syndrome

CamScanner ^ ^£>.,^11
14$
ii ijui y

Haemochromatosis

Mutations in the gene encoding the vasopressin receptor

Wolfram syndrome

Nephrogenic diabetes insipidus may be caused genetic


mutations:
• the more common form affects the vasopression ( ADH)
receptor
• the less common form results from a mutation in the gene
that encodes the aquaporin 2 channel

Importance: 50

The investigations show normal renal function, calcium and glycaemic


control. The 24-hour urine collection shows the patient is polyuric, and
the paired urine and serum osmolalities indicate the patient is
producing dilute urine. This is in keeping with a diagnosis of diabetes
insipidus. A water deprivation test will help distinguish if the cause is
nephrogenic or cranial.

Idiopathic, head injury, haemochromatosis and Wolfram's syndrome are


all cranial causes of diabetes insipidus.

Mutations in the gene that encode the vasopressin receptor are the only
nephrogenic cause listed.

B £ f # Discuss Improve

Next question >


CamScanner
A 48-year-old man known to have long-standing problems with alcohol
excess has been referred to hepatology with abdominal swelling. He is
diagnosed as having decompensated alcoholic liver disease with
ascites. The consultant starts him on spironolactone to help with the
management of his ascites.

What is the mechanism of action of this drug?

Inhibition of the sodium / potassium/ chloride transporter in the


loop of Henle

Inhibition of the sodium / chloride transporter in the distal


convoluted tubule

Inhibition of carbonic anhydrase in the proximal tubules

Inhibition of the epithelial sodium channel in the distal


convoluted tubule

Inhibition of the mineralocorticoid receptor in the cortical


collecting ducts

Submit answer

Reference ranges v

Score: 100%

1
2
3
CamScanner
ascites. The consultant starts him on spironolactone to help with the
management of his ascites.

What is the mechanism of action of this drug?

Inhibition of the sodium/ potassium/ chloride transporter in the loop


of Henle

Inhibition of the sodium / chloride transporter in the distal convoluted


tubule

Inhibition of carbonic anhydrase in the proximal tubules

Inhibition of the epithelial sodium channel in the distal convoluted


tubule

Inhibition of the mineralocorticoid receptor in the cortical collecting


ducts

Aldosterone antagonists acts on the cortical collecting ducts as a


diuretic
Important for me Less important

Spironolactone is an aldosterone antagonist, therefore, acts to inhibit


the mineralocorticoid receptor in the cortical collecting ducts.

Loop diuretics such as furosemide act to block the


sodium/ potassium/ chloride transporter in the loop of Henle.

Thiazide diuretics such as bendroflumethiazide block the


sodium/ chloride transporter in the distal convoluted tubules.

CamScanner
Carbonic anhydrase inhibitors include dorzolamide and act in the
proximal tubules.

Amiloride inhibits the epithelial sodium transporter in the distal


convoluted tubules.

BA A # Discuss Improve

Spironolactone

Spironolactone is an aldosterone antagonist which acts in the cortical


collecting duct.

Indications
• ascites: patients with cirrhosis develop a secondary
hyperaldosteronism. Relatively large doses such as 100 or 200mg
are often used
• hypertension: used in some patients as a NICE 'step 4' treatment
• heart failure (see RALES study below)
• nephrotic syndrome
• Conn's syndrome

Adverse effects
• hyperkalaemia
• gynaecomastia: less common with eplerenone

RALES
• NYHA III + IV, patients already taking ACE inhibitor
• low dose spironolactone reduces all cause mortality

B / a A =^ Tr H C-D

CamScanner

You might also like